Forum SFG Lvl-1 Test-5 @cse - Updates

You might also like

Download as pdf or txt
Download as pdf or txt
You are on page 1of 57

SFG 2023 |LEVEL 1 |Test 5 |

DO NOT OPEN THIS TEST BOOKLET UNTIL YOU ARE TOLD TO DO SO


T.B.C.: FIAS-SFG23-L1T5 Test Booklet Series
Serial No.

A
TEST BOOKLET
GENERAL STUDIES

Time Allowed: ONE HOUR Maximum Marks: 100


INSTRUCTIONS
1. IMMEDIATELY AFTER THE COMMENCEMENT OF THE EXAMINATION, YOU SHOULD CHECK
THAT THIS TEST BOOKLET DOES NOT HAVE ANY UNPRINTED OR TORN OR MISSING PAGES
OR ITEMS ETC. IF SO, GET IT REPLACED BY A COMPLETE TEST BOOK.
2. Please note that it is the candidate’s responsibility to encode and fill in the Roll Number and Test
Booklet Series A, B, C or D carefully without any omission or discrepancy at the appropriate
places in the OMR Answer Sheet. Any omission/discrepancy will render the Answer Sheet liable
for rejection.
3. You have to enter your Name, Email Id and
Name:
Mobile No. on the test booklet in the Box Email Id:
provided alongside. DO NOT write anything
Mobile No:
else on the Test Booklet.
4. This Test Booklet contains 50 items (questions). Each item is printed in English and Hindi. Each
item comprises four responses (answers). You will select the response which you want to mark
on the Answer Sheet. In case you feel that there is more than one correct response, mark the
response which you consider the best. In any case, choose ONLY ONE response for each item.
5. You have to mark all your responses ONLY on the separate Answer Sheet provided.
See directions in the Answer Sheet.
6. All items carry equal marks.
7. Before you proceed to mark in the Answer Sheet the response to various items in the Test Booklet,
you have to fill in some particulars in the Answer Sheet as per instructions sent to you with your
Admission Certificate.
8. After you have completed filling in all your responses on the Answer Sheet and the examination
has concluded, you should hand over to the Invigilator only the Answer Sheet. You are permitted
to take away with you the Test Booklet.
9. Sheets for rough work are appended in the Test Booklet at the end.
10. Penalty for wrong answers:
THERE WILL BE PENALTY FOR WRONG ANSWERS MARKED BY A CANDIDATE IN THE
OBJECTIVE TYPE QUESTION PAPERS.
(i) There are four alternatives for the answer to every question. For each question for which a
wrong answer has been given by the candidate, one-third of the marks assigned to that
question will be deducted as penalty.
(ii) If a candidate gives more than one answer, it will be treated as wrong answer even if one
of the given answers happens to be correct and there will be same penalty as above to that
question.
(iii) If a question is left bank i.e., no answer is given by the candidate, there will be no penalty
for that question.

Forum Learning Centre: Delhi - 2nd Floor, IAPL House, 19 Pusa Road, Karol Bagh, New Delhi - 110005 | Patna - 2nd floor, AG Palace, E Boring Canal
Road, Patna, Bihar 800001 | Hyderabad - 1st & 2nd Floor, SM Plaza, RTC X Rd, Indira Park Road, Jawahar Nagar, Hyderabad, Telangana 500020
9821711605 | https://academy.forumias.com | admissions@forumias.academy | helpdesk@forumias.academy
Page 1

SFG 2023 |LEVEL 1 |Test 5 |

Q.1) We adopted parliamentary democracy intervening clauses are


based on the British model, but how does our taken as passed.
model differ from that model? 4. Kangaroo undiscussed clauses of a
1. As regards legislation, the British Parliament Closure bill or a resolution are
is supreme or sovereign but in India, the also put to vote along
power of the Parliament to legislate is with the discussed ones
limited. How many pairs given above are correctly
2. In India, matters related to the matched?
constitutionality of the Amendment of an a) Only one pair
Act of the Parliament are referred to the b) Only two pairs
Constitution Bench by the Supreme Court. c) Only three pairs
Select the correct answer using the code given d) All four pairs
below.
a) 1 only Q.4) Which of the following provisions are
b) 2 only enshrined in the Indian Constitution to ensure
c) Both 1 and 2 independence and impartiality of the office of
d) Neither 1 nor 2 Lok Sabha Speaker?
1. S/he can be removed by a resolution passed
Q.2) With reference to ‘Zero Hour’, consider the by the Lok Sabha by an effective majority.
following statements: 2. S/he has not been given any voting power
1. In India, the proceedings of every in the Lok Sabha, except on his removal
parliamentary sitting generally start with motion.
Zero Hour. 3. After being appointed as Speaker, he/she
2. It allows the Members of the Parliament to resigns from his/her party to remain
raise important matters without any prior politically neutral.
notice. 4. The salaries and allowances of the Speaker
3. It is nowhere mentioned in the Rules of are charged on the Consolidated Fund of
Procedure of the Parliament. India.
Which of the statements given above is/are Select the correct answer from the code given
correct? below:
a) 3 only a) 1 and 2 only
b) 2 only b) 2 and 3 only
c) 2 and 3 only c) 1 and 4 only
d) 1, 2 and 3 d) 3 and 4 only

Q.3) Consider the following pairs regarding Q.5) Consider the following statements
Closure Motion: regarding ‘Lesser Florican’:
Types of Closure Description 1. They are endemic to Eastern African
Motion continent and fly to India only during
1. Simple Closure a member moves it when winters.
the ‘matter having been 2. The breeding and wintering sites of the bird
sufficiently discussed be requires sufficient grass cover.
now put to vote’ 3. It is one of the priority species for recovery
2. Closure by clauses of a bill or a under Integrated Development of Wildlife
Compartments lengthy resolution are Habitat scheme.
grouped into parts Which of the statements given above is/are
before the debate and correct?
the entire part is put to a) 1 and 2 only
vote b) 2 and 3 only
3. Guillotine only important clauses c) 3 only
Closure are taken up for debate d) 1, 2 and 3
and voting and the

Forum Learning Centre: Delhi - 2nd Floor, IAPL House, 19 Pusa Road, Karol Bagh, New Delhi - 110005 | Patna - 2nd floor, AG Palace, E Boring Canal
Road, Patna, Bihar 800001 | Hyderabad - 1st & 2nd Floor, SM Plaza, RTC X Rd, Indira Park Road, Jawahar Nagar, Hyderabad, Telangana 500020
9821711605 | https://academy.forumias.com | admissions@forumias.academy | helpdesk@forumias.academy
Page 2

SFG 2023 |LEVEL 1 |Test 5 |

Q.1) हमने ब्रिब्रिश मॉडल के आधार पर सं सदीय लोकतं त्र 4. कंगारू समापन ब्रकसी ब्रिधेयक या सं कल्प के
को अपनाया, लेब्रकन हमारा मॉडल उस मॉडल से कैसे अब्रिचाररत िंडों को भी चचाभ
अलग है ? के साथ मतदान के ब्रलए रिा
1. ब्रिधान के सं बंध में, ब्रिब्रिश सं सद सिोच्च या सं प्रभु है , जाता है
लेब्रकन भारत में सं सद की कानून बनाने की शक्ति ऊपर ब्रदए गए ब्रकतने यु ग् सही सु मेब्रलत हैं ?
सीब्रमत है । a) केिल एक यु ग्म
2. भारत में, सं सद के ब्रकसी अब्रधब्रनयम के सं शोधन की b) केिल दो यु ग्म
सं िैधाब्रनकता से सं बंब्रधत मामलों को सिोच्च न्यायालय c) केिल तीन यु ग्म
द्वारा संब्रिधान पीठ को भे जा जाता है । d) सभी चार यु ग्म
नीचे ब्रदए गए कूि का प्रयोग कर सही उत्तर चुब्रनए।
a) केिल 1 Q.4) लोकसभा अध्यक्ष के कायाभ लय की स्वतं त्रता और
b) केिल 2 ब्रनष्पक्षता सु ब्रनब्रित करने के ब्रलए भारतीय सं ब्रिधान में
c) 1 और 2 दोनों ब्रनम्नब्रलक्तित में से कौन से प्रािधान ब्रनब्रहत हैं ?
d) न तो 1 और न ही 2 1. उसे लोकसभा द्वारा प्रभािी बहुमत से पाररत प्रस्ताि
द्वारा हिाया जा सकता है ।
Q.2) 'शून्यकाल' के सं दभभ में, ब्रनम्नब्रलक्तित कथनों पर 2. उसे हिाने के प्रस्ताि को छोड़कर, उसे लोकसभा में
ब्रिचार करें : कोई मतदान शक्ति नही ं दी गई है ।
1. भारत में प्रत्येक सं सदीय बैठक की कायभ िाही 3. अध्यक्ष के रूप में ब्रनयु ि होने के बाद, िह
सामान्यतया शून्यकाल से शुरू होती है । राजनीब्रतक रूप से तिस्थ रहने के ब्रलए अपनी पािी
2. यह सं सद सदस्ों को ब्रबना ब्रकसी पूिभ सू चना के से इस्तीफा दे दे ता/दे ती है ।
महत्वपूर्भ मामले उठाने की अनु मब्रत दे ता है । 4. अध्यक्ष के िे तन और भत्ते भारत की सं ब्रचत ब्रनब्रध पर
3. सं सद के प्रब्रिया ब्रनयमों में इसका कही ं उल्ले ि नही ं भाररत होते हैं ।
है । नीचे ब्रदए गए कूि से सही उत्तर का चयन करें :
ऊपर ब्रदए गए कथनों में से कौन सा/से सही है /हैं ? a) केिल 1 और 2
a) केिल 3 b) केिल 2 और 3
b) केिल 2 c) केिल 1 और 4
c) केिल 2 और 3 d) केिल 3 और 4
d) 1, 2 और 3
Q.5) खरमोर (Lesser Florican) के सं बंध में ब्रनम्नब्रलक्तित
Q.3) समापन प्रस्ताव (Closure Motion) के सं बंध में कथनों पर ब्रिचार करें :
ब्रनम्नब्रलक्तित यु ग्ों पर ब्रिचार करें : 1. िे पूिी अफ्रीकी महाद्वीप के ब्रलए स्थाब्रनक हैं और
समापन प्रस्ताव के वििरण केिल सब्रदभयों के दौरान ही भारत आते हैं ।
प्रकार 2. पक्षी के प्रजनन और शीतकालीन स्थलों के ब्रलए
1. साधारण समापन एक सदस् इसे तब पेश करता पयाभ प्त घास के आिरर् की आिश्यकता होती है ।
है जब 'मामले पर पयाभ प्त चचाभ
3. यह िन्यजीि आिास योजना के एकीकृत ब्रिकास के
हो चुकी है और अब इसे
तहत पुनप्राभ क्तप्त के ब्रलए प्राथब्रमकता िाली प्रजाब्रतयों में
मतदान के ब्रलए रिा गया है '
2. खंडश: समापन ब्रकसी ब्रिधेयक के िंडों या से एक है ।
एक लंबे सं कल्प को बहस से ऊपर ब्रदए गए कथनों में से कौन सा/से सही है /हैं ?
पहले भागों में बां िा जाता है a) केिल 1 और 2
और पूरे भाग को मतदान के b) केिल 2 और 3
ब्रलए रिा जाता है c) केिल 3
3. गगलोगिन समापन बहस और मतदान के ब्रलए
d) 1, 2 और 3
केिल महत्वपूर्भ िंडों को
ब्रलया जाता है और बीच में
आने िाले िं डों को पाररत मान
ब्रलया जाता है ।

Forum Learning Centre: Delhi - 2nd Floor, IAPL House, 19 Pusa Road, Karol Bagh, New Delhi - 110005 | Patna - 2nd floor, AG Palace, E Boring Canal
Road, Patna, Bihar 800001 | Hyderabad - 1st & 2nd Floor, SM Plaza, RTC X Rd, Indira Park Road, Jawahar Nagar, Hyderabad, Telangana 500020
9821711605 | https://academy.forumias.com | admissions@forumias.academy | helpdesk@forumias.academy
Page 3

SFG 2023 |LEVEL 1 |Test 5 |

Q.6) Consider the following statements with 2. Unlike in Parliamentary system, President
reference to the Public Accounts Committee: cannot dissolve the lower house under
1. The committee has the equal number of Presidential system.
members from both Lok Sabha and Rajya 3. In both Parliamentary system and
Sabha. Presidential system, the governments can
2. The committee examines the annual audit be removed through a vote of no-
reports of the Comptroller and Auditor confidence.
General of India. Select the correct answer using the codes given
3. Its recommendations are binding on the below:
ministers. a) 1 and 2 only
4. Public account committee is concerned b) 1 only
with the questions of policy in broader c) 2 and 3 only
sense. d) 1, 2 and 3
Which of the statements given above is/are
correct?
Q.9) With reference to powers of Lok Sabha and
a) 2 only Rajya Sabha, consider the following statements:
b) 1 and 3 only 1. Both Lok Sabha and Rajya Sabha exercises
c) 2 and 3 only control over executive by introducing No-
d) 2 and 4 only confidence motion.
2. Under Indian Constitution, Rajya Sabha has
Q.7) Which of the following are among the no powers with regards to discontinuance
participants to elect the members to the of resolution of national emergency.
‘Legislative Council’ of a State? 3. Lok Sabha can reject all or any of the
1. Members of local bodies in the State. recommendations of the Rajya Sabha
2. All graduates of three years standing and regarding the Money Bill.
residing within the State. Which of the statements given above is/are
correct?
3. Members of the Legislative Assembly of the
State. a) 1 only
4. Members of Lok Sabha elected from the b) 1 and 3 only
State. c) 2 only
5. Members of Rajya Sabha nominated from d) 2 and 3 only
the State.
Select the correct answer using the code given Q.10) With reference to ‘Global Gateway Plan’,
below: consider the following statements:
a) 1, 2 and 3 only 1. The plan aims to invest globally in
b) 2, 4 and 5 only infrastructure, digital and climate projects.
c) 1, 2, 4 and 5 only 2. Global Gateway projects will be developed
d) 1, 3, 4 and 5 only and delivered through Team Europe
Initiatives under the European Commission.

Q.8) Which of the following statements is/are Which of the statements given above is/are
correct?
correct regarding the differences/similarities
between the Presidential form of Government a) 1 only
and the Parliamentary form of Government? b) 2 only
1. The Executive is responsible to the c) Both 1 and 2
legislature in only Parliamentary form of d) Neither 1 nor 2
government and not in Presidential form of
government.

Forum Learning Centre: Delhi - 2nd Floor, IAPL House, 19 Pusa Road, Karol Bagh, New Delhi - 110005 | Patna - 2nd floor, AG Palace, E Boring Canal
Road, Patna, Bihar 800001 | Hyderabad - 1st & 2nd Floor, SM Plaza, RTC X Rd, Indira Park Road, Jawahar Nagar, Hyderabad, Telangana 500020
9821711605 | https://academy.forumias.com | admissions@forumias.academy | helpdesk@forumias.academy
Page 4

SFG 2023 |LEVEL 1 |Test 5 |

Q.6) लोक लेिा सब्रमब्रत के सं दभभ में ब्रनम्नब्रलक्तित कथनों नीचे ब्रदए गए कूि का प्रयोग कर सही उत्तर चुब्रनए:
पर ब्रिचार करें : a) केिल 1 और 2
1. सब्रमब्रत में लोकसभा और राज्यसभा दोनों के सदस्ों b) केिल 1
की सं ख्या बराबर होती है ।
c) केिल 2 और 3
2. सब्रमब्रत भारत के ब्रनयं त्रक और महालेिा परीक्षक की
d) 1, 2 और 3
िाब्रषभक ऑब्रडि ररपोिभ की जां च करती है ।
3. इसकी ब्रसफाररशें मंब्रत्रयों के ब्रलए बाध्यकारी होती हैं ।
Q.9) लोकसभा और राज्यसभा की शक्तियों के सं दभभ में,
4. लोक लेिा सब्रमब्रत व्यापक अथों में नीब्रतगत प्रश्ों से
ब्रनम्नब्रलक्तित कथनों पर ब्रिचार करें :
सं बंब्रधत है ।
1. लोकसभा और राज्यसभा दोनों ही अब्रिश्वास प्रस्ताि
ऊपर ब्रदए गए कथनों में से कौन सा/से सही है /हैं ?
लाकर कायभ पाब्रलका पर ब्रनयं त्रर् रिती हैं ।
a) केिल 2
2. भारतीय संब्रिधान के तहत, राष्ट्रीय आपातकाल के
b) केिल 1 और 3 सं कल्प को रोकने के सं बंध में राज्य सभा के पास कोई
c) केिल 2 और 3 अब्रधकार नही ं है ।
d) केिल 2 और 4 3. लोकसभा धन ब्रिधे यक के सं बंध में राज्यसभा की सभी
या ब्रकसी भी ब्रसफाररश को अस्वीकार कर सकती है ।

Q.7) ब्रकसी राज्य की 'ब्रिधान पररषद' के सदस्ों का ऊपर ब्रदए गए कथनों में से कौन सा/से सही है /हैं ?
चुनाि करने िाले प्रब्रतभाब्रगयों में से कौन हैं ? a) केिल 1
1. राज्य में स्थानीय ब्रनकायों के सदस्। b) केिल 1 और 3
2. राज्य में स्थायी एिं ब्रनिास करने िाले तीन िषभ के सभी c) केिल 2
स्नातक। d) केिल 2 और 3
3. राज्य की ब्रिधान सभा के सदस्।
4. राज्य से ब्रनिाभ ब्रचत लोकसभा के सदस्। Q.10) ' ग्लोबल गे ििे प्लान ' के सं दभभ में , ब्रनम्नब्रलक्तित
5. राज्य सभा के सदस् राज्य से मनोनीत होते हैं । कथनों पर ब्रिचार करें :
नीचे ब्रदए गए कूि का प्रयोग कर सही उत्तर चुब्रनए: 1. योजना का उद्दे श्य बु ब्रनयादी ढां चे, ब्रडब्रजिल और
a) केिल 1, 2 और 3 जलिायु पररयोजनाओं में ब्रिश्व स्तर पर ब्रनिे श करना
है ।
b) केिल 2, 4 और 5
2. िै ब्रश्वक गे ििे पररयोजनाओं को यू रोपीय आयोग के
c) केिल 1, 2, 4 और 5
तहत िीम यूरोप पहल के माध्यम से ब्रिकब्रसत और
d) केिल 1, 3, 4 और 5
ब्रितररत ब्रकया जाएगा।
ऊपर ब्रदए गए कथनों में से कौन सा/से सही है /हैं ?
Q.8) सरकार के राष्ट्रपब्रत के रूप और सरकार के
a) केिल 1
सं सदीय स्वरूप के बीच मतभेदों / समानताओं के बारे में
b) केिल 2
ब्रनम्नब्रलक्तित में से कौन सा कथन सही है / हैं ?
c) 1 और 2 दोनों
1. कायभ पाब्रलका सरकार के केिल सं सदीय स्वरूप में
ब्रिधाब्रयका के प्रब्रत उत्तरदायी होती है न ब्रक d) न तो 1 और न ही 2
अध्यक्षात्मक सरकार में।
2. सं सदीय प्रर्ाली के ब्रिपरीत, राष्ट्रपब्रत राष्ट्रपब्रत प्रर्ाली
के तहत ब्रनचले सदन को भं ग नही ं कर सकता है ।
3. सं सदीय प्रर्ाली और राष्ट्रपब्रत प्रर्ाली दोनों में,
अब्रिश्वास प्रस्ताि के माध्यम से सरकारों को हिाया जा
सकता है .

Forum Learning Centre: Delhi - 2nd Floor, IAPL House, 19 Pusa Road, Karol Bagh, New Delhi - 110005 | Patna - 2nd floor, AG Palace, E Boring Canal
Road, Patna, Bihar 800001 | Hyderabad - 1st & 2nd Floor, SM Plaza, RTC X Rd, Indira Park Road, Jawahar Nagar, Hyderabad, Telangana 500020
9821711605 | https://academy.forumias.com | admissions@forumias.academy | helpdesk@forumias.academy
Page 5

SFG 2023 |LEVEL 1 |Test 5 |

Q.11) Consider the following statements: Q.14) In which of the following cases, a member
1. In India, there is no law restricting the of Parliament will be liable to be disqualified on
candidates from contesting in Lok Sabha the ground of defection according to the Tenth
election from three constituencies at a time. Schedule?
2. In 1991 Lok Sabha Election, Shri Devi Lal 1. If he/she voluntarily gives up membership
contested from three Lok Sabha of his party after being elected as Presiding
constituencies. officer of house.
3. As per the existing rules, if a candidate 2. If he/she votes contrary to any direction
contests in one Lok Sabha election from issued by his political party.
many constituencies, his/her party should 3. If he/she, as a nominated member, joins any
bear the cost of bye-elections to the political party within six months of taking
constituencies vacated by him/her in the his seat.
event of his/her winning in all the 4. If he/she, as an independent member joins
constituencies. any political party after being elected.
Which of the statements given above is/are Select the correct answer using the code given
correct? below:
a) 1 only a) 1 and 3 only
b) 2 only b) 2 only
c) 1 and 3 only c) 2 and 4 only
d) 2 and 3 only d) 2, 3 and 4 only

Q.12) With reference to ‘Money bills’, consider Q.15) Consider the following statements
the following statements: regarding ‘She is a Changemaker’ Programme:
1. They can be introduced either by a minister 1. It is a pan-India scheme that aims to impart
or by a private member. vocational training to women in various
2. They requires the certification of the technical crafts.
speaker whenever transmitted to the Rajya 2. It has been launched by the National
Sabha. Commission for Women.
3. Defeat of Money bills introduced in Lok Which of the statements given above is/are
Sabha leads to the resignation of the correct?
government. a) 1 only
Which of the statements given above is/are b) 2 only
correct? c) Both 1 and 2
a) 1 only d) Neither 1 nor 2
b) 1 and 2 only
c) 3 only Q.16) A Joint Session of Parliament can be called
d) 2 and 3 only for which of the following types of bills?
1. Money bill
Q.13) What are the ways through which 2. Ordinary bill
Parliament controls the executive? 3. Financial bills (I)
1. Not passing Motion of Thanks 4. Constitutional Amendment bills under
2. Passing of cut motion Article 368
3. Anti-defection law Select the correct answer using the code given
4. Half-an-hour discussion below:
5. Removal of Deputy-speaker of Lok Sabha a) 1, 2 and 3 only
Select the correct answer using the code given b) 2, 3 and 4 only
below: c) 2 and 3 only
a) 1 and 3 only d) 2 only
b) 1, 2 and 4 only
c) 2, 3 and 4 only
d) 1, 3 and 5 only

Forum Learning Centre: Delhi - 2nd Floor, IAPL House, 19 Pusa Road, Karol Bagh, New Delhi - 110005 | Patna - 2nd floor, AG Palace, E Boring Canal
Road, Patna, Bihar 800001 | Hyderabad - 1st & 2nd Floor, SM Plaza, RTC X Rd, Indira Park Road, Jawahar Nagar, Hyderabad, Telangana 500020
9821711605 | https://academy.forumias.com | admissions@forumias.academy | helpdesk@forumias.academy
Page 6

SFG 2023 |LEVEL 1 |Test 5 |

Q.11) ब्रनम्नब्रलक्तित कथनों पर ब्रिचार करें : Q.14) ब्रनम्नब्रलक्तित में से ब्रकस मामले में, दसिीं अनुसूची
1. भारत में, एक समय में तीन ब्रनिाभ चन क्षे त्रों से लोकसभा के अनुसार सं सद के एक सदस् को दलबदल के आधार
चुनाि लड़ने से उम्मीदिारों को प्रब्रतबं ब्रधत करने िाला पर अयोग्य घोब्रषत ब्रकया जा सकता है ?
कोई कानून नही ं है । 1. यब्रद िह सदन के पीठासीन अब्रधकारी के रूप में चुने
2. 1991 के लोकसभा चुनाि में, श्री दे िीलाल ने तीन जाने के बाद स्वेच्छा से अपनी पािी की सदस्ता छोड़
लोकसभा क्षे त्रों से चुनाि लड़े थे। दे ता है ।
3. मौजूदा गनयमों के अनुसार, यगद कोई उम्मीदवार कई 2. यब्रद िह अपने राजनीब्रतक दल द्वारा जारी ब्रकए गए
गनवाा चन क्षे त्ों से एक लोकसभा चुनाव लड़ता है , तो ब्रकसी ब्रनदे श के ब्रिपरीत मतदान करता/करती है ।
उसकी पािी को सभी गनवाा चन क्षे त्ों में जीतने की 3. यब्रद िह मनोनीत सदस् के रूप में अपनी सीि लेने
स्थथगत में उसके द्वारा खाली गकए गए गनवाा चन क्षे त्ों के के छह महीने के भीतर ब्रकसी राजनीब्रतक दल में
उपचु नावों का खचा वहन करना चागहए। शाब्रमल हो जाता है ।
ऊपर ब्रदए गए कथनों में से कौन सा/से सही है /हैं ? 4. यब्रद िह एक स्वतं त्र सदस् के रूप में चुने जाने के
a) केिल 1 बाद ब्रकसी राजनीब्रतक दल में शाब्रमल होता/होती है ।
b) केिल 2 नीचे ब्रदए गए कूि का प्रयोग कर सही उत्तर चुब्रनए:
c) केिल 1 और 3 a) केिल 1 और 3
d) केिल 2 और 3 b)केिल 2
c) केिल 2 और 4
Q.12) 'धन गवधे यक' के सं दभभ में, ब्रनम्नब्रलक्तित कथनों पर d) केिल 2, 3 और 4
ब्रिचार करें :
1. उसको या तो ब्रकसी मंत्री या ब्रनजी सदस् द्वारा पेश Q.15) शी इज ए चेंजमेकर' (She is a Changemaker)
गकया जा सकता है । कायभ िम के सं बंध में ब्रनम्नब्रलक्तित कथनों पर ब्रिचार करें :
2. राज्य सभा को भे जे जाने पर उन्हें अध्यक्ष के 1. यह एक अक्तिल भारतीय योजना है ब्रजसका उद्दे श्य
प्रमार्ीकरर् की आिश्यकता होती है । ब्रिब्रभन्न तकनीकी ब्रशल्पों में मब्रहलाओं को
3. लोकसभा में पेश ब्रकए गए धन ब्रिधे यकों की हार से व्यािसाब्रयक प्रब्रशक्षर् प्रदान करना है ।
सरकार को इस्तीफा दे ना पड़ता है । 2. इसे राष्ट्रीय मब्रहला आयोग द्वारा शुरू ब्रकया गया है ।
ऊपर ब्रदए गए कथनों में से कौन सा/से सही है /हैं ? ऊपर ब्रदए गए कथनों में से कौन सा/से सही है /हैं ?
a) केिल 1 a) केिल 1
b) केिल 1 और 2 b) केिल 2
c) केिल 3 c) 1 और 2 दोनों
d) केिल 2 और 3 d) न तो 1 और न ही 2

Q.13) सं सद ब्रकन तरीकों से कायभ पाब्रलका को ब्रनयं ब्रत्रत Q.16) ब्रनम्नब्रलक्तित में से ब्रकस प्रकार के ब्रिधे यकों के ब्रलए
करती है ? सं सद का सं युि सत्र बु लाया जा सकता है ?
1. धन्यिाद प्रस्ताि पाररत नही ं करना 1. धन ब्रिधे यक
2. किौती प्रस्ताव पास नही ं करना 2. साधारर् गवधे यक
3. दल-बदल ब्रिरोधी कानून 3. ब्रित्तीय गवधे यक (I)
4. आधे घंिे की चचाभ 4. अनुच्छेद 368 के तहत सं ब्रिधान सं शोधन ब्रिधे यक
5. लोक सभा के उपसभापब्रत को हिाना नीचे ब्रदए गए कूि का प्रयोग कर सही उत्तर चुब्रनए:
नीचे ब्रदए गए कूि का प्रयोग कर सही उत्तर चुब्रनए: a) केिल 1, 2 और 3
a) केिल 1 और 3 b) केिल 2, 3 और 4
b) केिल 1, 2 और 4 c) केिल 2 और 3
c) केिल 2, 3 और 4 d) केिल 2
d) केिल 1, 3 और 5

Forum Learning Centre: Delhi - 2nd Floor, IAPL House, 19 Pusa Road, Karol Bagh, New Delhi - 110005 | Patna - 2nd floor, AG Palace, E Boring Canal
Road, Patna, Bihar 800001 | Hyderabad - 1st & 2nd Floor, SM Plaza, RTC X Rd, Indira Park Road, Jawahar Nagar, Hyderabad, Telangana 500020
9821711605 | https://academy.forumias.com | admissions@forumias.academy | helpdesk@forumias.academy
Page 7

SFG 2023 |LEVEL 1 |Test 5 |

Q.17) With reference to Rajya Sabha, consider Q.20) With reference to ‘Socio-Economic
the following statements: Policies of States’, which one of the following
1. Rajya Sabha can authorise the Parliament to statements best describes the term ‘New
create new All-India Services. Welfarism”?
2. A resolution for the removal of the Vice- a) It is a policy of prioritising the supply of
President can be introduced only in the public goods such as basic health and
Rajya Sabha. primary education through top-down
approach.
Which of the statements given above is/are
b) It is a form of neo-liberal policy that seeks
correct? for greater role of private sector in welfare
a) 1 only sectors like education and health.
b) 2 only c) It focuses on the role of state in providing
c) Both 1 and 2 subsidized tangible essential goods and
d) Neither 1 nor 2 services such as bank accounts, cooking gas,
toilets, electricity etc.
d) It is an approach that entails direct
Q.18) With reference to Parliamentary Elections
monetary benefits to the targeted group
after Independence, consider the following instead of providing physical support
statements: system.
1. The highest number of seats ever in any
election to Lok Sabha was won by the Janta Q.21) In India, which of the following reviews
party in 1984. the independent regulators in sectors like
2. Seventeenth Lok Sabha of 2019 has the telecommunications, insurance, electricity,
highest percentage of women Member of etc.?
Parliaments since Independence. 1. Ad Hoc Committee set up by the Parliament.
3. The 10th Lok Sabha election of 1991 2. Parliamentary Department Related Standing
recorded the highest voter participations Committee
since independence. 3. Finance Commission
4. The first ever coalition government in 4. Financial Sector Legislative Reforms
Centre was formed by Communist Party of Commission
India (M) in 1977. 5. NITI Aayog
Which of the statement given above is/are Select the correct answer using the code given
correct? below.
a) 1 and 2 only a) 1 and 2 only
b) 2 only b) 1, 3 and 4 only
c) 1, 2 and 4 only c) 3, 4 and 5 only
d) 3 and 4 only d) 2 and 5 only

Q.22) With reference to the method of


Q.19) Arrange the following items in the correct
Proportional Representation for elections,
order of the passing of an ordinary bill in the
consider the following statements:
Parliament:
1. The system of Territorial Representation
1. Consideration Stage
represents the electorate more effectively
2. Stage of General Discussion as compared to Proportional
3. Third Reading Representation system.
4. Assent of the President 2. The List System of Proportional
5. Publication of the bill in the Gazette Representation is adopted for the election
Select the correct answer from the code given of the Rajya Sabha members.
below: 3. This system does not give any scope for
organising by-elections.
a) 1-2-3-4-5
Which of the statements given above is/are
b) 2-1-3-4-5
correct?
c) 5-2-1-3-4
a) 1 only
d) 5-1-2-3-4
b) 3 only
c) 1 and 2 only
d) 2 and 3 only

Forum Learning Centre: Delhi - 2nd Floor, IAPL House, 19 Pusa Road, Karol Bagh, New Delhi - 110005 | Patna - 2nd floor, AG Palace, E Boring Canal
Road, Patna, Bihar 800001 | Hyderabad - 1st & 2nd Floor, SM Plaza, RTC X Rd, Indira Park Road, Jawahar Nagar, Hyderabad, Telangana 500020
9821711605 | https://academy.forumias.com | admissions@forumias.academy | helpdesk@forumias.academy
Page 8

SFG 2023 |LEVEL 1 |Test 5 |

Q.17) राज्यसभा के सं दभभ में, ब्रनम्नब्रलक्तित कथनों पर Q.20) ' राज्यों की सामाब्रजक-आब्रथभक नीब्रतयों ' के सं दभभ
ब्रिचार करें : में, ब्रनम्नब्रलक्तित में से कौन सा कथन ' नि कल्यार्िाद '
1. राज्य सभा नई अक्तिल भारतीय से िाओं के सृ जन के शब्द का सबसे अच्छा िर्भ न करता है ?
ब्रलए सं सद को अब्रधकृत कर सकती है । a) यह िॉप-डाउन दृब्रष्ट्कोर् के माध्यम से बु ब्रनयादी
2. उपराष्ट्रपब्रत को पद से हिाने का प्रस्ताि केिल राज्य स्वास्थ्य और प्राथब्रमक ब्रशक्षा जैसी सािभ जब्रनक िस्तु ओं
सभा में ही पेश ब्रकया जा सकता है । की आपूब्रतभ को प्राथब्रमकता दे ने की नीब्रत है ।

ऊपर ब्रदए गए कथनों में से कौन सा/से सही है /हैं ? b) यह नि-कल्याणवाद नीब्रत का एक रूप है जो ब्रशक्षा
और स्वास्थ्य जैसे कल्यार्कारी क्षे त्रों में ब्रनजी क्षेत्र की
a) केिल 1
अब्रधक भू ब्रमका की मां ग करता है।
b) केिल 2
c) मूतभ आिश्यक िस्तु ओं और से िाओं को प्रदान करने
c) 1 और 2 दोनों
में राज्य की भू ब्रमका पर केंब्रद्रत है जैसे बैं क िाते ,
d) न तो 1 और न ही 2 रसोई गै स, शौचालय, ब्रबजली आब्रद।
d) यह एक दृब्रष्ट्कोर् है जो भौब्रतक सहायता प्रर्ाली
Q.18) स्वतं त्रता के बाद सं सदीय चुनािों के सं दभभ में, प्रदान करने के बजाय लब्रक्षत समूह को प्रत्यक्ष मौब्रद्रक
ब्रनम्नब्रलक्तित कथनों पर ब्रिचार करें : लाभ प्रदान करता है ।
1. जनता पािी ने 1984 में लोकसभा के ब्रकसी भी चुनाि
में अब तक की सबसे अब्रधक सीिें जीती थी।ं Q.21) भारत में ब्रनम्नब्रलक्तित में से कौन दू रसं चार, बीमा,
2. 2019 की सत्रहिी ं लोकसभा में आजादी के बाद से ब्रबजली आब्रद जैसे क्षे त्रों में स्वतंत्र ब्रनयामकों की समीक्षा
सबसे अब्रधक मब्रहला सां सदों का प्रब्रतशत है । करता है ?
3. 1991 के 10िें लोकसभा चुनाि में आजादी के बाद से 1. सं सद द्वारा गब्रठत तदथभ सब्रमब्रत।
सिाभ ब्रधक मतदाता भागीदारी दजभ की गई। 2. सं सदीय ब्रिभाग सं बंधी स्थायी सब्रमब्रत
4. केंद्र में पहली बार गठबं धन सरकार का गठन भारतीय 3. ब्रित्त आयोग
कम्युब्रनस्ट पािी (M) द्वारा 1977 में ब्रकया गया था। 4. ब्रित्तीय क्षे त्र ब्रिधायी सु धार आयोग
ऊपर ब्रदए गए कथनों में से कौन सा/से सही है /हैं ? 5. नीब्रत आयोग

a) केिल 1 और 2 नीचे ब्रदए गए कूि का प्रयोग कर सही उत्तर चुब्रनए।

b) केिल 2 a) केिल 1 और 2
b) केिल 1, 3 और 4
c) केिल 1, 2 और 4
c) केिल 3, 4 और 5
d) केिल 3 और 4
d) केिल 2 और 5

Q.19) सं सद में एक साधारर् ब्रिधे यक के पाररत होने के Q.22) चुनािों के ब्रलए आनुपाब्रतक प्रब्रतब्रनब्रधत्व की पद्धब्रत
सही िम में ब्रनम्नब्रलक्तित मदों को व्यिक्तस्थत करें : के सं दभभ में, ब्रनम्नब्रलक्तित कथनों पर ब्रिचार करें :
1. गवचार की चरण 1. प्रादे ब्रशक प्रब्रतब्रनब्रधत्व प्रर्ाली आनुपाब्रतक प्रब्रतब्रनब्रधत्व
2. सामान्य चचाभ का चरर् प्रर्ाली की तु लना में अब्रधक प्रभािी ढं ग से मतदाताओं
का प्रब्रतब्रनब्रधत्व करती है ।
3. तृ तीय वचन 2. राज्यसभा सदस्ों के चुनाि के ब्रलए आनुपाब्रतक
4. राष्ट्रपब्रत की स्वीकृब्रत प्रब्रतब्रनब्रधत्व की सू ची प्रर्ाली अपनाई जाती है ।
3. यह प्रर्ाली उपचुनािों के आयोजन की कोई गुं जाइश
5. राजपत्र में ब्रिधे यक का प्रकाशन
नही ं दे ती है .
नीचे ब्रदए गए कूि से सही उत्तर का चयन करें : ऊपर ब्रदए गए कथनों में से कौन सा/से सही है /हैं ?
a)1-2-3-4-5 a) केिल 1
b) 2-1-3-4-5 b) केिल 3
c) 5-2-1-3-4 c) केिल 1 और 2
d) 5-1-2-3-4 d) केिल 2 और 3

Forum Learning Centre: Delhi - 2nd Floor, IAPL House, 19 Pusa Road, Karol Bagh, New Delhi - 110005 | Patna - 2nd floor, AG Palace, E Boring Canal
Road, Patna, Bihar 800001 | Hyderabad - 1st & 2nd Floor, SM Plaza, RTC X Rd, Indira Park Road, Jawahar Nagar, Hyderabad, Telangana 500020
9821711605 | https://academy.forumias.com | admissions@forumias.academy | helpdesk@forumias.academy
Page 9

SFG 2023 |LEVEL 1 |Test 5 |

Q.23) With reference to ‘Delimitation 2. The Act of Parliament to create a Legislative


Commission’ in India, consider the following Council is considered a Constitutional
statements: Amendment under Article 368.
1. The orders of the Commission cannot be 3. The actual strength of the Legislative
called in question before any court. Council in a State is fixed by the Governor of
2. Modifications to orders of the Commission that State.
can only be done by the Lok Sabha and not 4. The retiring members of the Council are
by State Legislative Assemblies. eligible for re-election and re-nomination
Which of the statements given above is/are for any number of times.
correct? Which of the Statements given above is/are
a) 1 only correct?
b) 2 only a) 2 and 3 only
c) Both 1 and 2 b) 4 only
d) Neither 1 nor 2 c) 3 and 4 only
d) 1, 2, 3 only
Q.24) In India, President is an integral part of
the Parliament. In this context, consider the Q.27) With reference to the Anglo-Indians,
following statements: consider the following statements:
1. A bill passed by both the houses of 1. The term ‘Anglo-Indian’ is defined in the
Parliament cannot become law without the Indian Constitution
President’s assent. 2. The original Indian Constitution of 1950
2. S/he is the Leader of the House of the Lok provided for the reservation for Anglo-
Sabha. Indians in Lok Sabha.
3. S/he presides over joint sitting of two 3. The 104th Constitutional Amendment Act,
Houses of Parliament. 2020 discontinued the reservation of Anglo-
Which of the statements given above is/are Indians in the Lok Sabha.
correct? Which of the statements given above are
a) 1 only correct?
b) 2 and 3 only a) 1 and 2 only
c) 1 and 2 only b) 2 and 3 only
d) 1 and 3 only c) 1 and 3 only
d) 1, 2 and 3
Q.25) Consider the following statements with
reference to Public Key Cryptography: Q.28) Which of the following States/Union
1. It is a communication where people Territories in India have bicameral legislature?
exchange messages that can only be read by 1. Puducherry
one another. 2. Andhra Pradesh
2. Data that is encrypted with the public key 3. Uttar Pradesh
can be decrypted only with the 4. Rajasthan
corresponding private key. 5. Bihar
Which of the statements given above is/are 6. Karnataka
correct? 7. Jammu & Kashmir
a) 1 only Select the correct answer using the code given
b) 2 only below:
c) Both 1 and 2 a) 1, 2, 3, 5 and 7 only
d) Neither 1 nor 2 b) 2, 3, 5 and 6 only
c) 2, 3, 4, 5 and 6 only
Q.26) With reference to the Legislative Council d) 1, 2, 3, 5 and 6 only
in a State, consider the following Statements:
1. A Legislative Council can be created by the
Parliament only when Rajya Sabha passes a
resolution to that effect.

Forum Learning Centre: Delhi - 2nd Floor, IAPL House, 19 Pusa Road, Karol Bagh, New Delhi - 110005 | Patna - 2nd floor, AG Palace, E Boring Canal
Road, Patna, Bihar 800001 | Hyderabad - 1st & 2nd Floor, SM Plaza, RTC X Rd, Indira Park Road, Jawahar Nagar, Hyderabad, Telangana 500020
9821711605 | https://academy.forumias.com | admissions@forumias.academy | helpdesk@forumias.academy
Page 10

SFG 2023 |LEVEL 1 |Test 5 |

Q.23) भारत में 'पररसीमन आयोग' के सं दभभ में, 2. ब्रिधान पररषद बनाने के सं सद के अब्रधब्रनयम को
ब्रनम्नब्रलक्तित कथनों पर ब्रिचार करें : अनु च्छेद 368 के तहत एक सं िैधाब्रनक सं शोधन माना
1. आयोग के आदे शों को ब्रकसी न्यायालय के समक्ष जाता है ।
प्रश्गत नही ं ब्रकया जा सकता है । 3. ब्रकसी राज्य में ब्रिधान पररषद की िास्तब्रिक सं ख्या
2. आयोग के आदे शों में सं शोधन केिल लोकसभा द्वारा उस राज्य के राज्यपाल द्वारा तय की जाती है ।
ब्रकया जा सकता है न ब्रक राज्य ब्रिधानसभाओं द्वारा। 4. पररषद के से िाब्रनिृ त्त सदस् ब्रकतनी भी बार
ऊपर ब्रदए गए कथनों में से कौन सा/से सही है /हैं ? पुनब्रनभिाभ चन और पुननाभ मां कन के ब्रलए पात्र होते हैं ।
a) केिल 1 ऊपर ब्रदए गए कथनों में से कौन-सा/से सही है /हैं ?
b) केिल 2 a) केिल 2 और 3
c) 1 और 2 दोनों b) केिल 4
d) न तो 1 और न ही 2 c) केिल 3 और 4
d) केिल 1, 2, 3
Q.24) भारत में, राष्ट्रपब्रत सं सद का एक अब्रभन्न अंग है ।
इस सं दभभ में ब्रनम्नब्रलक्तित कथनों पर ब्रिचार कीब्रजएः Q.27) एं ग्लो-इं ब्रडयन के सं दभभ में, ब्रनम्नब्रलक्तित कथनों पर
1. सं सद के दोनों सदनों द्वारा पाररत ब्रिधे यक राष्ट्रपब्रत की ब्रिचार करें :
सहमब्रत के ब्रबना कानून नही ं बन सकता। 1. 'एं ग्लो-इं ब्रडयन' शब्द को भारतीय संब्रिधान में
2. िह लोकसभा के सदन का नेता होता है । पररभाब्रषत ब्रकया गया है
3. िह सं सद के दोनों सदनों की सं युि बै ठक की 2. 1950 के मूल भारतीय संब्रिधान ने लोकसभा में एं ग्लो-
अध्यक्षता करता है । इं ब्रडयन के ब्रलए आरक्षर् प्रदान ब्रकया।
ऊपर ब्रदए गए कथनों में से कौन सा/से सही है /हैं ? 3. 104िें सं ब्रिधान सं शोधन अब्रधब्रनयम, 2020 ने
a) केिल 1 लोकसभा में एं ग्लो-इं ब्रडयन के आरक्षर् को समाप्त
b) केिल 2 और 3 कर ब्रदया।
c) केिल 1 और 2 ऊपर ब्रदए गए कथनों में से कौन से सही हैं ?
d) केिल 1 और 3 a) केिल 1 और 2
b) केिल 2 और 3
Q.25) सािभ जब्रनक कुंजी ब्रिप्टोग्राफी के सं दभभ में c) केिल 1 और 3
ब्रनम्नब्रलक्तित कथनों पर ब्रिचार करें : d) 1, 2 और 3
1. यह एक सं चार है जहााँ लोग सं देशों का आदान-प्रदान
करते हैं ब्रजन्हें केिल एक दू सरे द्वारा पढा जा सकता Q.28) भारत में ब्रनम्नब्रलक्तित में से ब्रकन राज्यों / केंद्र
है । शाब्रसत प्रदे शों में ब्रद्वसदनीय ब्रिधाब्रयका है ?
2. सािभ जब्रनक कुंजी से एक्तरिप्ट ब्रकया गया डे िा केिल 1. पुडुचेरी
सं बंब्रधत ब्रनजी कुंजी के साथ ब्रडब्रिप्ट ब्रकया जा सकता 2. आं ध्र प्रदे श
है । 3. उत्तर प्रदे श
ऊपर ब्रदए गए कथनों में से कौन सा/से सही है /हैं ? 4. राजस्थान
a) केिल 1 5. ब्रबहार
b) केिल 2 6. कनाभ िक
c) 1 और 2 दोनों 7. जम्मू और कश्मीर
d) न तो 1 और न ही 2 नीचे ब्रदए गए कूि का प्रयोग कर सही उत्तर चुब्रनए:
a) केिल 1, 2, 3, 5 और 7
Q.26) ब्रकसी राज्य में ब्रिधान पररषद के सं दभभ में, b) केिल 2, 3, 5 और 6
ब्रनम्नब्रलक्तित कथनों पर ब्रिचार करें : c) केिल 2, 3, 4, 5 और 6
1. सं सद द्वारा एक ब्रिधान पररषद तभी बनाई जा सकती d) केिल 1, 2, 3, 5 और 6
है जब राज्य सभा उस आशय का प्रस्ताि पाररत करे ।

Forum Learning Centre: Delhi - 2nd Floor, IAPL House, 19 Pusa Road, Karol Bagh, New Delhi - 110005 | Patna - 2nd floor, AG Palace, E Boring Canal
Road, Patna, Bihar 800001 | Hyderabad - 1st & 2nd Floor, SM Plaza, RTC X Rd, Indira Park Road, Jawahar Nagar, Hyderabad, Telangana 500020
9821711605 | https://academy.forumias.com | admissions@forumias.academy | helpdesk@forumias.academy
Page 11

SFG 2023 |LEVEL 1 |Test 5 |

Q.29) With reference to the position of 1. One of its functions is to ensure payment of
Legislative Council of a State and the Rajya salary and other allowances to the Members
Sabha under the Indian Constitution, consider of the House.
the following Statements: 2. It is headed by Minister of Parliamentary
1. Like the Rajya Sabha, the Legislative Affairs.
Councils cannot reject or amend a Money 3. The recruitment and service conditions of
Bill. the secretarial staff are regulated by the
2. Unlike the Rajya Sabha, the Legislative Council of Ministers.
Council does not participate in the election Which of the statements given above is/are
of the President of India. correct?
3. The Legislative Council does not have any a) 1 only
exclusive or special powers as enjoyed by b) 2 only
the Rajya Sabha. c) 1 and 3 only
Which of the Statements given above are d) 2 and 3 only
correct?
a) 1 and 2 only Q.33) With reference to the Parliament in India,
b) 2 and 3 only consider the following statements:
c) 1 and 3 only 1. A bill passed by the Rajya Sabha but pending
d) 1, 2 and 3 in the Lok Sabha does not lapse on the
dissolution of the Lok Sabha
Q.30) Consider the following statements with 2. A bill passed by the Lok Sabha but pending
reference to Magnetars: in the Rajya Sabha does not lapse on the
1. A Magnetar is a type of neutron star that has dissolution of the Lok Sabha.
an ultra-powerful magnetic field. 3. Prorogation of the House does not affect the
2. Magnetars can be more luminous than Sun bills pending before the House.
even in an inactive state. Which of the statements given above is/are
Which of the statements given above is/are correct?
correct? a) 1 and 3 only
a) 1 only b) 3 only
b) 2 only c) 1 and 2 only
d) 2 and 3 only
c) Both 1 and 2
d) Neither 1 nor 2
Q.34) With reference to Indian Parliament,
Consider the following statements about
Q.31) Out of the following statements, choose
Department Related Standing Committees
the one that brings out the principle underlying
(DRSCs):
the Cabinet form of Government:
a) An arrangement for minimizing the 1. They are constitutional bodies mentioned in
criticism against the Government whose Part V of the Constitution of India.
responsibilities are complex and hard to 2. The term of office of each standing
carry out the satisfaction of all.
committee is coterminous with the life of
b) A mechanism for speeding up the activities
of the Government whose responsibilities Lok Sabha.
are increasing day by day. 3. They secure accountability of the Council of
c) A mechanism of parliamentary democracy Ministers towards the Parliament.
for ensuring collective responsibility of the
Government to the people. 4. They assist the Parliament in scrutinizing
d) A device for strengthening the hands of the the bills referred to it.
head of the Government whose hold over Which of the statements given above is/are
the people is in a state of decline. correct?
a) 2, 3 and 4 only
Q.32) With reference to the Secretariat of
b) 3 only
Parliament in India, consider the following
statements: c) 3 and 4 only
d) 1, 2, 3 and 4

Forum Learning Centre: Delhi - 2nd Floor, IAPL House, 19 Pusa Road, Karol Bagh, New Delhi - 110005 | Patna - 2nd floor, AG Palace, E Boring Canal
Road, Patna, Bihar 800001 | Hyderabad - 1st & 2nd Floor, SM Plaza, RTC X Rd, Indira Park Road, Jawahar Nagar, Hyderabad, Telangana 500020
9821711605 | https://academy.forumias.com | admissions@forumias.academy | helpdesk@forumias.academy
Page 12

SFG 2023 |LEVEL 1 |Test 5 |

Q.29) भारतीय संब्रिधान के तहत ब्रकसी राज्य की ब्रिधान 1. इसका एक कायभ सदन के सदस्ों को िे तन और अन्य
पररषद और राज्य सभा की क्तस्थब्रत के सं दभभ में, भत्तों का भु गतान सु ब्रनब्रित करना है ।
ब्रनम्नब्रलक्तित कथनों पर ब्रिचार करें : 2. इसकी अध्यक्षता सं सदीय कायभ मंत्री करते हैं ।
1. राज्य सभा की तरह, ब्रिधान पररषदें धन ब्रिधे यक को 3. सब्रचिीय कमभचाररयों की भती और से िा शतों को
अस्वीकार या सं शोब्रधत नही ं कर सकती हैं । मंब्रत्रपररषद द्वारा ब्रिब्रनयब्रमत ब्रकया जाता है ।
2. राज्य सभा के ब्रिपरीत, ब्रिधान पररषद भारत के ऊपर ब्रदए गए कथनों में से कौन सा/से सही है /हैं ?
राष्ट्रपब्रत के चुनाि में भाग नही ं लेती है । a) केिल 1
3. ब्रिधान पररषद के पास राज्य सभा की तरह कोई b) केिल 2
अनन्य या ब्रिशेष शक्तियां नही ं हैं । c) केिल 1 और 3
ऊपर ब्रदए गए कथनों में से कौन से सही हैं ? d) केिल 2 और 3
a) केिल 1 और 2
b) केिल 2 और 3 Q.33) भारत में सं सद के सं दभभ में, ब्रनम्नब्रलक्तित कथनों
c) केिल 1 और 3 पर ब्रिचार करें :
d) 1, 2 और 3 1. राज्य सभा द्वारा पाररत लेब्रकन लोकसभा में लंब्रबत
ब्रिधे यक लोकसभा के ब्रिघिन पर व्यपगत नही ं होता
Q.30) मै ग्नेटर्स के सं दभभ में ब्रनम्नब्रलक्तित कथनों पर ब्रिचार है
करें : 2. लोकसभा द्वारा पाररत लेब्रकन राज्य सभा में लंब्रबत
1. एक मैग्नेिसभ एक प्रकार का न्यूिरॉन तारा है ब्रजसमें एक ब्रिधे यक लोकसभा के ब्रिघिन पर समाप्त नही ं होता
अब्रत-शक्तिशाली चुंबकीय क्षे त्र होता है । है ।
2. मैग्नेिसभ ब्रनक्तिय अिस्था में भी सू यभ से अब्रधक 3. सदन का सत्रािसान सदन के समक्ष लंब्रबत ब्रिधे यकों
चमकदार हो सकते हैं । को प्रभाब्रित नही ं करता है ।
ऊपर ब्रदए गए कथनों में से कौन सा/से सही है /हैं ? ऊपर ब्रदए गए कथनों में से कौन सा/से सही है /हैं ?
a) केिल 1 a) केिल 1 और 3
b) केिल 2 b) केिल 3
c) 1 और 2 दोनों c) केिल 1 और 2
d) न तो 1 और न ही 2 d) केिल 2 और 3

Q.31) ब्रनम्नब्रलक्तित कथनों में से , उस एक को चुनें जो Q.34) भारतीय सं सद के सं दभभ में, ब्रिभाग सं बंब्रधत स्थायी
सरकार के कैब्रबनेि रूप में अंतब्रनभब्रहत ब्रसद्धां त को सामने सब्रमब्रतयों (DRSCs) के बारे में ब्रनम्नब्रलक्तित कथनों पर
लाता है : ब्रिचार करें :
a) सरकार के क्तिलाफ आलोचना को कम करने की 1. िे भारत के सं ब्रिधान के भाग V में उक्तल्लक्तित
व्यिस्था ब्रजसकी ब्रजम्मेदाररयां जब्रिल हैं और सभी की सं िैधाब्रनक ब्रनकाय हैं ।
सं तुब्रष्ट् को पूरा करना कब्रठन है । 2. प्रत्येक स्थायी सब्रमब्रत के कायभकाल लोकसभा के
b) सरकार की गब्रतब्रिब्रधयों को ते ज करने के ब्रलए एक कायभ काल तक सीब्रमत होता है ।
तं त्र ब्रजसकी ब्रजम्मेदाररयां ब्रदन-ब-ब्रदन बढती जा रही 3. िे सं सद के प्रब्रत मंब्रत्रपररषद की जिाबदे ही सु ब्रनब्रित
हैं । करते हैं ।
c) लोगों के प्रब्रत सरकार की सामूब्रहक ब्रजम्मेदारी 4. िे सं सद को सं दब्रभभत ब्रिधे यकों की जां च करने में
सु ब्रनब्रित करने के ब्रलए सं सदीय लोकतं त्र का एक तं त्र। सहायता करते हैं ।
d) सरकार के प्रमुि के हाथों को मजबू त करने के ब्रलए ऊपर ब्रदए गए कथनों में से कौन सा/से सही है /हैं ?
एक उपकरर् ब्रजसकी जनता पर पकड़ कम हो रही a) केिल 2, 3 और 4
है । b) केिल 3
c) केिल 3 और 4
Q.32) भारत में सं सद के सब्रचिालय के सं दभभ में, d) 1, 2, 3 और 4
ब्रनम्नब्रलक्तित कथनों पर ब्रिचार करें :

Forum Learning Centre: Delhi - 2nd Floor, IAPL House, 19 Pusa Road, Karol Bagh, New Delhi - 110005 | Patna - 2nd floor, AG Palace, E Boring Canal
Road, Patna, Bihar 800001 | Hyderabad - 1st & 2nd Floor, SM Plaza, RTC X Rd, Indira Park Road, Jawahar Nagar, Hyderabad, Telangana 500020
9821711605 | https://academy.forumias.com | admissions@forumias.academy | helpdesk@forumias.academy
Page 13

SFG 2023 |LEVEL 1 |Test 5 |

Q.35) Which among the following statements Which of the statements given above is/are
is/are correct with regards to Type 1 Diabetes? correct?
1. It accounts for the vast majority (over 90%) a) 1 and 2 only
of diabetes worldwide. b) 2 only
2. It is the major type of diabetes in childhood c) 1 and 3 only
but can occur at any age.
d) None of the above
Select the correct answer using the code given
below:
Q.38) Consider the following statements with
a) 1 only
reference to the Constitutional provisions
b) 2 only regarding the Budget:
c) Both 1 and 2 1. Parliament does not have right either to
d) Neither 1 nor 2 discuss or to vote on the expenditures
charged on the Consolidated Fund of India.

Q.36) Which of the following pairs are correctly 2. The term ‘budget’ has nowhere been used in
matched? the Constitution of India.
3. With regard to the enactment of budget,
List I List II
Parliament can reduce or abolish a tax but
Cut motions Description cannot increase it.
1. Policy Cut The amount of the demand 4. Vote on the demands for grants is an
Motion be reduced by ₹100 exclusive privilege of the Lok Sabha and
Rajya Sabha has no power regarding it.
2. Token Cut The amount of the demand
Motion be reduced to Rs 1 Which of the statements given above are
correct?
3. Economy Cut The amount of the demand
a) 1, 2 and 3 only
Motion be reduced by a specified
amount b) 2, 3 and 4 only
c) 1, 3 and 4 only
Select the correct answer using the code given
below: d) 1, 2, 3 and 4

a) 1 only
b) 2 and 3 only Q.39) With reference to Private Member’s Bills
and Public Bills, Consider the following
c) 3 only
statements:
d) 1 and 2 only
1. Both types of bills can be introduced in the
Parliament after only seven days' notice.
Q.37) Consider the following statements with 2. No private member’s bill has been passed by
reference to the Joint Sitting of the Parliament: Parliament of India till date.
1. It is governed by the Rules of Procedure of 3. A bill introduced by an elected member of
both Lok Sabha and Rajya Sabha. Parliament is called as public bill, while a bill
2. No new amendments can be proposed to a introduced by a nominated member is called
bill at a joint sitting. as private bill.
3. The bill is deemed to have been passed only Which of the statements given above is/are
if it is passed by a majority of the total incorrect?
membership of both the houses of
a) 1 and 2 only
parliament.
b) 2 and 3 only
c) 1 only
d) 1, 2 and 3

Forum Learning Centre: Delhi - 2nd Floor, IAPL House, 19 Pusa Road, Karol Bagh, New Delhi - 110005 | Patna - 2nd floor, AG Palace, E Boring Canal
Road, Patna, Bihar 800001 | Hyderabad - 1st & 2nd Floor, SM Plaza, RTC X Rd, Indira Park Road, Jawahar Nagar, Hyderabad, Telangana 500020
9821711605 | https://academy.forumias.com | admissions@forumias.academy | helpdesk@forumias.academy
Page 14

SFG 2023 |LEVEL 1 |Test 5 |

Q.35) िाइप 1 मधु मेह के सं बंध में ब्रनम्नब्रलक्तित में से कौन ऊपर ब्रदए गए कथनों में से कौन सा/से सही है /हैं ?
सा/से कथन सही है /हैं ? a) केिल 1 और 2
1. यह दु ब्रनया भर में मधु मेह के अगधक सं क्रमण (90% b) केिल 2
से अब्रधक) के ब्रलए ब्रजम्मेदार है ।
c) केिल 1 और 3
2. यह बचपन में मधु मेह का प्रमुि प्रकार है लेब्रकन यह
d) उपरोि में से कोई नही ं
ब्रकसी भी उम्र में हो सकता है ।
नीचे ब्रदए गए कूि का प्रयोग कर सही उत्तर चुब्रनए:
Q.38) बजि के सं बंध में सं िैधाब्रनक प्रािधानों के सं दभभ
a) केिल 1
में ब्रनम्नब्रलक्तित कथनों पर ब्रिचार करें :
b) केिल 2
1. सं सद को भारत की संब्रचत ब्रनब्रध पर भाररत व्यय पर
c) 1 और 2 दोनों चचाभ करने या मतदान करने का अब्रधकार नही ं है ।
d) न तो 1 और न ही 2 2. भारत के सं ब्रिधान में कही ं भी 'बजि' शब्द का प्रयोग
नही ं ब्रकया गया है ।
Q.36) ब्रनम्नब्रलक्तित में से कौन से जोड़े सही ढं ग से मेल 3. बजि के अब्रधब्रनयमन के संबंध में, सं सद ब्रकसी कर
िाते हैं ? को घिा या समाप्त कर सकती है , लेब्रकन बढा नही ं
सकती।
र्ूची I र्ूची वितीय
4. अनुदान की मां गों पर मतदान लोकसभा का एक
कट मोशन वििरण
ब्रिशेष ब्रिशेषाब्रधकार है और राज्यसभा के पास इसके
1. नीब्रत किौती मां ग की राब्रश ₹ 100 कम की बारे में कोई शक्ति नही ं है ।
प्रस्ताि जाए
ऊपर ब्रदए गए कथनों में से कौन से सही हैं ?
2. सां केगतक किौती मां ग की राब्रश घिाकर 1 रुपये a) केिल 1, 2 और 3
प्रस्ताव कर दी जाए
b) केिल 2, 3 और 4
3. आगथाक किौती मां ग की राब्रश को एक ब्रनब्रदभष्ट् c) केिल 1, 3 और 4
प्रस्ताि राब्रश से कम ब्रकया जाना
d) 1, 2, 3 और 4
चाब्रहए

नीचे ब्रदए गए कूि का प्रयोग कर सही उत्तर चुब्रनए: Q.39) ब्रनजी सदस्ों के ब्रिधेयकों और सािभ जब्रनक
a) केिल 1 ब्रिधे यकों के संदभभ में, ब्रनम्नब्रलक्तित कथनों पर ब्रिचार करें :
b) केिल 2 और 3 1. दोनों प्रकार के ब्रिधे यकों को केिल सात ब्रदनों के
c) केिल 3 नोब्रिस के बाद सं सद में पेश ब्रकया जा सकता है ।

d) केिल 1 और 2 2. भारत की सं सद द्वारा आज तक कोई भी ब्रनजी सदस्


ब्रिधे यक पाररत नही ं ब्रकया गया है।
3. सं सद के ब्रनिाभ ब्रचत सदस् द्वारा पेश ब्रकए गए ब्रिधे यक
Q.37) सं सद की सं युि बै ठक के सं दभभ में ब्रनम्नब्रलक्तित
को सािभ जब्रनक ब्रिधे यक कहा जाता है , जबब्रक नाब्रमत
कथनों पर ब्रिचार करें :
सदस् द्वारा पेश ब्रकए गए ब्रिधे यक को ब्रनजी ब्रिधे यक
1. यह लोक सभा और राज्य सभा दोनों के प्रब्रिया
कहा जाता है ।
ब्रनयमों द्वारा शाब्रसत होता है ।
ऊपर ब्रदए गए कथनों में से कौन सा/से गलत है /हैं ?
2. सं युि बै ठक में ब्रकसी ब्रिधे यक में कोई नया सं शोधन
a) केिल 1 और 2
प्रस्ताब्रित नही ं ब्रकया जा सकता है।
b) केिल 2 और 3
3. ब्रिधे यक को तभी पाररत माना जाता है जब यह सं सद
के दोनों सदनों की कुल सदस्ता के बहुमत से पाररत c) केिल 1
हो जाता है । d) 1, 2 और 3

Forum Learning Centre: Delhi - 2nd Floor, IAPL House, 19 Pusa Road, Karol Bagh, New Delhi - 110005 | Patna - 2nd floor, AG Palace, E Boring Canal
Road, Patna, Bihar 800001 | Hyderabad - 1st & 2nd Floor, SM Plaza, RTC X Rd, Indira Park Road, Jawahar Nagar, Hyderabad, Telangana 500020
9821711605 | https://academy.forumias.com | admissions@forumias.academy | helpdesk@forumias.academy
Page 15

SFG 2023 |LEVEL 1 |Test 5 |

Q.40) Consider the following pairs in context of Q.43) With reference to the parliamentary
places in news: committees in parliamentary system of
government, Consider the following
Place in news Country/Region
statements:
1. Barbados Caribbean Islands 1. The Constitution of India makes no mention
2. Donbas Poland of parliamentary committees.
2. Parliamentary committees provide a forum
3. Amami Islands China
for building consensus across political
Which of the pairs given above is/are correctly parties.
matched? 3. Consultative Committees are a type of
a) 1 only Parliamentary committees.
b) 1 and 2 only Which of the statements given above is/are
c) 1 and 3 only correct?
d) 2 and 3 only a) 1 and 2 only
b) 2 only
c) 2 and 3 only
Q.41) Consider the following statements:
d) 1, 2 and 3
1. In the election for Lok Sabha or State
Assembly, the winning candidate must get at
least 50 percent of the votes polled, to be Q.44) The origin of this committee can be
declared elected. traced back to the standing financial committee
set up in 1921. It was first constituted in the
2. According to the provisions laid down in the
post-independence era on the
Constitution of India, in Lok Sabha, the
recommendation of John Mathai. Originally the
Speaker’s post goes to the majority party
committee had 25 members but in 1956 its
and the Deputy Speaker’s to the Opposition.
membership was raised to 30. All the members
Which of the statements given above is/are
of this committee are from Lok Sabha only. The
correct?
Rajya Sabha has no representation in this
a) 1 only committee.
b) 2 only a) Public Accounts Committee
c) Both 1 and 2
b) Estimates Committee
d) Neither 1 nor 2
c) Committee on Public Undertakings
d) Business Advisory Committee
Q.42) Consider the following statements with
reference to the finance bill and appropriation
Q.45) Which among the following statements
bill:
is/are incorrect with regards to Media
1. A Finance Bill is a Money Bill.
Regulation in India:
2. The government of India cannot withdraw 1. Press Council of India is assigned with the
money from the Consolidated Fund of India authority to regulate all kind of media
till the enactment of the appropriation bill. coverage including Print, Radio and TV
3. Unlike the Appropriation Bill, the news.
amendments seeking to reject or reduce a 2. The News Broadcasters and Digital
tax can be moved in the case of finance bill. Association (NBDA) is a self-regulatory body
Which of the statements given above is/are for the digital broadcasters.
correct? Select the correct answer using the code given
below:
a) 1 and 2 only
a) 1 only
b) 2 only
b) 2 only
c) 1 and 3 only c) Both 1 and 2
d) 1, 2 and 3 d) Neither 1 nor 2

Forum Learning Centre: Delhi - 2nd Floor, IAPL House, 19 Pusa Road, Karol Bagh, New Delhi - 110005 | Patna - 2nd floor, AG Palace, E Boring Canal
Road, Patna, Bihar 800001 | Hyderabad - 1st & 2nd Floor, SM Plaza, RTC X Rd, Indira Park Road, Jawahar Nagar, Hyderabad, Telangana 500020
9821711605 | https://academy.forumias.com | admissions@forumias.academy | helpdesk@forumias.academy
Page 16

SFG 2023 |LEVEL 1 |Test 5 |

Q.40) समाचारों में स्थानों के संदभभ में ब्रनम्नब्रलक्तित यु ग्ों Q.43) सरकार की सं सदीय प्रर्ाली में सं सदीय सब्रमब्रतयों
पर ब्रिचार करें : के सं दभभ में, ब्रनम्नब्रलक्तित कथनों पर ब्रिचार करें :
खबरोों में स्थान दे श/क्षेत्र 1. भारत के सं ब्रिधान में सं सदीय सब्रमब्रतयों का कोई
उल्ले ि नही ं है ।
1. बारबाडोस कैरे ब्रबयन द्वीप समूह
2. सं सदीय सब्रमब्रतयााँ राजनीब्रतक दलों के बीच आम
2. डोनबास पोलैंड
सहमब्रत बनाने के ब्रलए एक मं च प्रदान करती हैं ।
3. अमामी द्वीप चीन
3. सलाहकार सब्रमब्रतयााँ एक प्रकार की सं सदीय
ऊपर ब्रदए गए यु ग्ों में से कौन-सा/से सही सु मेब्रलत है /हैं ? सब्रमब्रतयााँ होती हैं ।
a) केिल 1 ऊपर ब्रदए गए कथनों में से कौन सा/से सही है /हैं ?
b) केिल 1 और 2 a) केिल 1 और 2
c) केिल 1 और 3 b) केिल 2
d) केिल 2 और 3 c) केिल 2 और 3
d) 1, 2 और 3
Q.41) ब्रनम्नब्रलक्तित कथनों पर ब्रिचार करें :
1. लोकसभा या राज्य ब्रिधानसभा के चुनाि में, जीतने Q.44) इस सब्रमब्रत की उत्पब्रत्त का पता 1921 में स्थाब्रपत
िाले उम्मीदिार को ब्रनिाभ ब्रचत घोब्रषत होने के ब्रलए स्थायी ब्रित्तीय सब्रमब्रत से लगाया जा सकता है । इसे पहली
डाले गए मतों का कम से कम 50 प्रब्रतशत प्राप्त करना बार जॉन मथाई की ब्रसफाररश पर स्वतं त्रता के बाद के यु ग
चाब्रहए। में गब्रठत ब्रकया गया था। मूल रूप से सब्रमब्रत में 25 सदस्
2. भारत के सं ब्रिधान में ब्रनधाभ ररत प्रािधानों के अनुसार थे लेब्रकन 1956 में इसकी सदस्ता बढाकर 30 कर दी
लोकसभा में अध्यक्ष का पद बहुमत िाली पािी को गई। इस सब्रमब्रत के सभी सदस् लोकसभा से ही हैं । इस
और उपाध्यक्ष का पद ब्रिपक्ष को होता है । सब्रमब्रत में राज्यसभा का कोई प्रब्रतब्रनब्रधत्व नही ं है ।
ऊपर ब्रदए गए कथनों में से कौन सा/से सही है /हैं ? a) लोक लेिा सब्रमब्रत
a) केिल 1 b) प्राक्कलन सब्रमब्रत
b) केिल 2 c) सािभ जब्रनक उपिमों पर सब्रमब्रत
c) 1 और 2 दोनों d) व्यापार सलाहकार सब्रमब्रत
d) न तो 1 और न ही 2
Q.45) भारत में मीब्रडया ब्रिब्रनयमन के सं बंध में
Q.42) ब्रित्त ब्रिधे यक और ब्रिब्रनयोग ब्रिधे यक के सं दभभ में ब्रनम्नब्रलक्तित में से कौन सा/से कथन गलत है /हैं :
ब्रनम्नब्रलक्तित कथनों पर ब्रिचार करें : 1. प्रेस काउं ब्रसल ऑफ इं ब्रडया को ब्रप्रंि, रे ब्रडयो और िीिी
1. ब्रित्त ब्रिधे यक धन ब्रिधे यक होता है । समाचार सब्रहत सभी प्रकार के मीब्रडया किरे ज को

2. भारत सरकार ब्रिब्रनयोग ब्रिधे यक के अब्रधब्रनयब्रमत ब्रिब्रनयब्रमत करने का अब्रधकार ब्रदया गया है ।

होने तक भारत की सं ब्रचत ब्रनब्रध से धन की ब्रनकासी 2. न्यूज िॉडकास्टसभ एं ड ब्रडब्रजिल एसोब्रसएशन


नही ं कर सकती है । (NBDA) ब्रडब्रजिल प्रसारकों के ब्रलए एक स्व-

3. ब्रिब्रनयोग ब्रिधे यक के ब्रिपरीत, ब्रित्त ब्रिधे यक के मामले ब्रनयामक ब्रनकाय है ।

में कर को अस्वीकार करने या कम करने की मां ग नीचे ब्रदए गए कूि का प्रयोग कर सही उत्तर चुब्रनए:
करने िाले सं शोधनों को स्थानां तररत ब्रकया जा सकता a) केिल 1
है । b) केिल 2
ऊपर ब्रदए गए कथनों में से कौन सा/से सही है /हैं ? c) 1 और 2 दोनों
a) केिल 1 और 2 d) न तो 1 और न ही 2
b) केिल 2
c) केिल 1 और 3
d) 1, 2 और 3

Forum Learning Centre: Delhi - 2nd Floor, IAPL House, 19 Pusa Road, Karol Bagh, New Delhi - 110005 | Patna - 2nd floor, AG Palace, E Boring Canal
Road, Patna, Bihar 800001 | Hyderabad - 1st & 2nd Floor, SM Plaza, RTC X Rd, Indira Park Road, Jawahar Nagar, Hyderabad, Telangana 500020
9821711605 | https://academy.forumias.com | admissions@forumias.academy | helpdesk@forumias.academy
Page 17

SFG 2023 |LEVEL 1 |Test 5 |

Q.46) Consider the following statements with Q.49) Consider the following statements with
reference to the Consultative Committees: respect to Parliamentary Forums in India:
1. These committees are headed by a member 1. They are constituted at the start of Budget
of parliament other than a minister. Session each year.
2. These committees provide a forum for 2. They seek to provide a platform to the
informal discussions between the ministers Members of Parliament to have interactions
and the members of Parliament on policies with subject experts on a matter.
of the government. 3. The Speaker of Lok Sabha is the ex-officio
3. Consultative committees are constituted by Vice-President of all the parliamentary
the Ministry of Home Affairs. forums.
Which of the statements given above is/are Which of the statements given above is/are
correct? correct?
a) 2 only a) 1 only
b) 2 and 3 only b) 2 and 3 only
c) 1 and 3 only c) 2 only
d) 1 and 2 d) 1, 2 and 3

Q.47) What are the basic functions of Q.50) Which one of the following
Parliament in India? agencies/organizations has released the
1. Exercising control over the Council of National Multidimensional Poverty Index
Ministers (MPI)?
2. Implementation of the Laws a) Department of Health and Research
3. Final interpretation of Constitution b) Oxfam India
4. Amending various provisions of the c) United Nations Development Programme
Constitution (UNDP)
5. Enactment of Annual Financial Statement of d) NITI Aayog
Government
Select the correct answer using the code given
below:
a) 1, 2 and 5 only
b) 3, 4 and 5 only
c) 1, 4 and 5 only
d) 1, 2, 3, 4 and 5

Q.48) Consider the following statements with


reference to the Contingency Fund of India:
1. Any expenditure incurred from this fund
does not require any kind of authorization
from the Parliament.
2. The Corpus of this fund has been fixed by
the Constitution.
3. The Cabinet Secretary holds the fund on
behalf of the President of India.
Which of the statements given above is/are
correct?
a) 1 and 2 only
b) 2 only
c) 1 and 3 only
d) None

Forum Learning Centre: Delhi - 2nd Floor, IAPL House, 19 Pusa Road, Karol Bagh, New Delhi - 110005 | Patna - 2nd floor, AG Palace, E Boring Canal
Road, Patna, Bihar 800001 | Hyderabad - 1st & 2nd Floor, SM Plaza, RTC X Rd, Indira Park Road, Jawahar Nagar, Hyderabad, Telangana 500020
9821711605 | https://academy.forumias.com | admissions@forumias.academy | helpdesk@forumias.academy
Page 18

SFG 2023 |LEVEL 1 |Test 5 |

Q.46) परामशादात्ी सब्रमब्रतयों के सं दभभ में ब्रनम्नब्रलक्तित Q.49) भारत में सं सदीय मंचों के सं बंध में ब्रनम्नब्रलक्तित
कथनों पर ब्रिचार करें : कथनों पर ब्रिचार करें :
1. इन सब्रमब्रतयों की अध्यक्षता एक मंत्री के अलािा अन्य 1. इनका गठन प्रत्येक िषभ बजि सत्र के प्रारं भ में ब्रकया
सं सद सदस् द्वारा की जाती है । जाता है ।
2. ये सब्रमब्रतयााँ सरकार की नीब्रतयों पर मंब्रत्रयों और सं सद 2. िे ब्रकसी मामले पर ब्रिषय ब्रिशे षज्ों के साथ सं वाद
के सदस्ों के बीच अनौपचाररक चचाभ के ब्रलए एक करने के ब्रलए सं सद सदस्ों को एक मंच प्रदान करना
मंच प्रदान करती हैं । चाहते हैं ।
3. गृ ह मंत्रालय द्वारा परामशभदात्री सब्रमब्रतयों का गठन 3. लोकसभा अध्यक्ष सभी सं सदीय मंचों का पदे न
ब्रकया जाता है । उपाध्यक्ष होता है ।
ऊपर ब्रदए गए कथनों में से कौन सा/से सही है /हैं ? ऊपर ब्रदए गए कथनों में से कौन सा/से सही है /हैं ?
a) केिल 2 a) केिल 1
b) केिल 2 और 3 b) केिल 2 और 3
c) केिल 1 और 3 c) केिल 2
d) 1 और 2 d) 1, 2 और 3

Q.47) भारत में सं सद के मूल कायभ क्या हैं ? Q.50) ब्रनम्नब्रलक्तित में से ब्रकस एजेंसी / सं गठन ने राष्ट्रीय
1. मंब्रत्रपररषद पर ब्रनयं त्रर् रिना बहुआयामी गरीबी सू चकां क (MPI) जारी करता है ?
2. कानूनों का कायाभ न्वयन a) स्वास्थ्य और अनुसंधान ब्रिभाग
3. सं ब्रिधान की अंब्रतम व्याख्या b) ऑक्सफैम इं ब्रडया
4. सं ब्रिधान के ब्रिब्रभन्न प्रािधानों में सं शोधन c) सं युि राष्ट्र ब्रिकास कायभ िम (UNDP)
5. सरकार के िाब्रषभक ब्रित्तीय ब्रििरर् का अब्रधब्रनयमन d) नीब्रत आयोग
नीचे ब्रदए गए कूि का प्रयोग कर सही उत्तर चुब्रनए:
a) केिल 1, 2 और 5
b) केिल 3, 4 और 5
c) केिल 1, 4 और 5
d) 1, 2, 3, 4 और 5

Q.48) भारत की आकक्तिकता ब्रनब्रध के सं दभभ में


ब्रनम्नब्रलक्तित कथनों पर ब्रिचार करें :
1. इस कोष से होने िाले ब्रकसी भी िचभ के ब्रलए सं सद से
ब्रकसी प्रकार के प्राब्रधकरर् की आिश्यकता नही ं होती
है ।
2. इस कोष का गवत्त संब्रिधान द्वारा ब्रनधाभ ररत ब्रकया गया
है ।
3. कैब्रबनेि सब्रचि भारत के राष्ट्रपब्रत की ओर से फंड
रिता है ।
ऊपर ब्रदए गए कथनों में से कौन सा/से सही है /हैं ?
a) केिल 1 और 2
b) केिल 2
c) केिल 1 और 3
d) कोई नही ं

Forum Learning Centre: Delhi - 2nd Floor, IAPL House, 19 Pusa Road, Karol Bagh, New Delhi - 110005 | Patna - 2nd floor, AG Palace, E Boring Canal
Road, Patna, Bihar 800001 | Hyderabad - 1st & 2nd Floor, SM Plaza, RTC X Rd, Indira Park Road, Jawahar Nagar, Hyderabad, Telangana 500020
9821711605 | https://academy.forumias.com | admissions@forumias.academy | helpdesk@forumias.academy
Page 1 of 38

SFG 2023 | LEVEL 1 | Test #5 – Solutions |


Q.1) We adopted parliamentary democracy based on the British model, but how does our model differ from
that model?
1. As regards legislation, the British Parliament is supreme or sovereign but in India, the power of the
Parliament to legislate is limited.
2. In India, matters related to the constitutionality of the Amendment of an Act of the Parliament are
referred to the Constitution Bench by the Supreme Court.
Select the correct answer using the code given below.
a) 1 only
b) 2 only
c) Both 1 and 2
d) Neither 1 nor 2

Ans) c
Exp) Option c is the correct answer.
Statement 1 is correct. Parliamentary sovereignty is a principle of the UK constitution. It makes
Parliament the supreme legal authority in the UK, which can create or end any law. Generally, the courts
cannot overrule its legislation and no Parliament can pass laws that future Parliaments cannot change.
Indian Parliament is not a sovereign body like the British Parliament. The Indian Parliament may, in
exercise of its constituent power, amend by way of addition, variation or repeal any provision of the
Constitution in accordance with the procedure laid down for the purpose. However, the Parliament cannot
amend those provisions which form the ‘basic structure’ of the Constitution. This was ruled by the Supreme
Court in the Kesavananda Bharati case (1973).
Statement 2 is correct. In India, matters related to the constitutionality of an amendment of an act of the
Parliament are referred to the Constitution Bench by the Supreme Court. A Constitution Bench is a bench
of the Supreme Court having five or more judges on it. These benches are not a routine phenomenon. A
vast majority of cases before the Supreme Court are heard and decided by a bench of two judges (called a
Division Bench), and sometimes of three. Constitution Benches are set up when the case involves a
substantial question of law pertaining to the interpretation of the Constitution (Article 145(3) of the
Constitution, which mandates that such matters be heard by a bench of not less than five judges).
Presently, Constitution Benches are set up on an ad hoc basis as and when the need arises. The idea behind
a Constitution Bench is clear: it is constituted in rare cases to decide important questions of fact or legal
and/or constitutional interpretation.
Source) M Laxmikanth, 6th edition, Chapter 3 Salient features of the Constitution\
https://www.barandbench.com/columns/constituting-constitution-benches-of-the-supreme-court-
an-analysis
UPSC CSE 2021

Q.2) With reference to ‘Zero Hour’, consider the following statements:


1. In India, the proceedings of every parliamentary sitting generally start with Zero Hour.
2. It allows the Members of the Parliament to raise important matters without any prior notice.
3. It is nowhere mentioned in the Rules of Procedure of the Parliament.
Which of the statements given above is/are correct?
a) 3 only
b) 2 only
c) 2 and 3 only
d) 1, 2 and 3

Forum Learning Centre: Delhi - 2nd Floor, IAPL House, 19 Pusa Road, Karol Bagh, New Delhi - 110005 | Patna - 2nd floor, AG Palace, E Boring Canal Road,
Patna, Bihar 800001 | Hyderabad - 1st & 2nd Floor, SM Plaza, RTC X Rd, Indira Park Road, Jawahar Nagar, Hyderabad, Telangana 500020
9821711605 | https://academy.forumias.com | admissions@forumias.academy | helpdesk@forumias.academy
Page 2 of 38

SFG 2023 | LEVEL 1 | Test #5 – Solutions |


Ans) c
Exp) Option c is the correct answer.
Statement 1 is incorrect: The first hour of every parliamentary sitting is slotted for Question hour. During
this time, the members ask questions and the ministers usually give answers. Zero hour is the time gap
between the question hour and the agenda. The zero hour starts immediately after the question hour and
lasts until the agenda for the day is taken up.
Statement 2 is correct: Zero Hour is an informal device available to the members of the Parliament to raise
matters without any prior notice.
Statement 3 is correct: Unlike the question hour, the zero hour is not mentioned in the Rules of
Procedure. It is an Indian innovation in the field of parliamentary procedures and has been in existence
since 1962.
Source: No. 12. Short Duration Discussions.pmd (loksabhaph.nic.in)
Questions: Lok Sabha
M LAXMIKANT

Q.3) Consider the following pairs regarding Closure Motion:


Types of Closure Description
Motion
1. Simple Closure a member moves it when
the ‘matter having been
sufficiently discussed be
now put to vote’
2. Closure by clauses of a bill or a
Compartments lengthy resolution are
grouped into parts before
the debate and the entire
part is put to vote
3. Guillotine Closure only important clauses
are taken up for debate
and voting and the
intervening clauses are
taken as passed.
4. Kangaroo Closure undiscussed clauses of a
bill or a resolution are
also put to vote along with
the discussed ones
How many pairs given above are correctly matched?
a) Only one pair
b) Only two pairs
c) Only three pairs
d) All four pairs

Ans) b
Exp) Option b is the correct answer.
Closure motion is moved by a member to cut short the debate on a matter before the House. If the motion
is approved by the House, debate is stopped forthwith and the matter is put to vote.
Pair 1 is correctly matched: Simple Closure is closing debate when a member moves it when the ‘matter
having been sufficiently discussed be now put to vote’.

Forum Learning Centre: Delhi - 2nd Floor, IAPL House, 19 Pusa Road, Karol Bagh, New Delhi - 110005 | Patna - 2nd floor, AG Palace, E Boring Canal Road,
Patna, Bihar 800001 | Hyderabad - 1st & 2nd Floor, SM Plaza, RTC X Rd, Indira Park Road, Jawahar Nagar, Hyderabad, Telangana 500020
9821711605 | https://academy.forumias.com | admissions@forumias.academy | helpdesk@forumias.academy
Page 3 of 38

SFG 2023 | LEVEL 1 | Test #5 – Solutions |


Pair 2 is correctly matched: Closure by Compartment happens when the clauses of a bill or a lengthy
resolution are grouped into parts before the commencement of the debate. The debate covers the part as
a whole and the entire part is put to vote.
Pair 3 is incorrectly matched: Guillotine Closure is one when the undiscussed clauses of a bill or a
resolution are also put to vote along with the discussed ones due to want of time (as the time allotted for
the discussion is over).
Pair 4 is incorrectly matched: Kangaroo Closure happens only when important clauses are taken up for
debate and voting and the intervening clauses are skipped over and taken as passed.
Source: M laxmikant (Chap- 22: Parliament)

Q.4) Which of the following provisions are enshrined in the Indian Constitution to ensure independence
and impartiality of the office of Lok Sabha Speaker?
1. S/he can be removed by a resolution passed by the Lok Sabha by an effective majority.
2. S/he has not been given any voting power in the Lok Sabha, except on his removal motion.
3. After being appointed as Speaker, he/she resigns from his/her party to remain politically neutral.
4. The salaries and allowances of the Speaker are charged on the Consolidated Fund of India.
Select the correct answer from the code given below:
a) 1 and 2 only
b) 2 and 3 only
c) 1 and 4 only
d) 3 and 4 only

Ans) c
Exp) Option c is the correct answer.
As the office of the Speaker is vested with great prestige, position and authority, independence and
impartiality become its sine qua non. A number of safeguards are taken to ensure the independence and
impartiality of the office of the Speaker.
Statement 1 is correct: The Speaker is provided with a security of tenure. He can be removed only by a
resolution passed by the Lok Sabha by an effective majority (i.e., a majority of all the then members of the
House) and not by an ordinary majority (i.e., a majority of the members present and voting in the House).
This motion of removal can be considered and discussed only when it has the support of at least 50
members.
Statement 2 is incorrect: The Speaker cannot vote on a bill in the Lok Sabha in the first instance. He can
exercise a casting vote in the event of a tie. This makes the position of Speaker impartial.
Statement 3 is incorrect: In Britain, the Speaker is strictly a nonparty man. There is a convention that the
Speaker has to resign from his party and remain politically neutral. This healthy convention is not fully
established in India where the Speaker does not resign from the membership of his party on his election
to the exalted office.
Statement 4 is correct: The salaries and allowances of the Speaker are fixed by Parliament, and they are
charged on the Consolidated Fund of India (I.e., not subjected to the Parliamentary approval).
Knowledge Base:
Other provisions to ensure the independence and impartiality of the office of the Speaker:
1) His salaries and allowances are fixed by Parliament. They are charged on the Consolidated Fund of India
and thus are not subject to the annual vote of Parliament.
2) His work and conduct cannot be discussed and criticised in the Lok Sabha except on a substantive
motion.

Forum Learning Centre: Delhi - 2nd Floor, IAPL House, 19 Pusa Road, Karol Bagh, New Delhi - 110005 | Patna - 2nd floor, AG Palace, E Boring Canal Road,
Patna, Bihar 800001 | Hyderabad - 1st & 2nd Floor, SM Plaza, RTC X Rd, Indira Park Road, Jawahar Nagar, Hyderabad, Telangana 500020
9821711605 | https://academy.forumias.com | admissions@forumias.academy | helpdesk@forumias.academy
Page 4 of 38

SFG 2023 | LEVEL 1 | Test #5 – Solutions |


3) His powers of regulating procedure or conducting business or maintaining order in the House are not
subject to the jurisdiction of any Court.
Source: Indian Polity by Laxmikant – 6th Edition – Chapter 22 Parliament.

Q.5) Consider the following statements regarding ‘Lesser Florican’:


1. They are endemic to Eastern African continent and fly to India only during winters.
2. The breeding and wintering sites of the bird requires sufficient grass cover.
3. It is one of the priority species for recovery under Integrated Development of Wildlife Habitat scheme.
Which of the statements given above is/are correct?
a) 1 and 2 only
b) 2 and 3 only
c) 3 only
d) 1, 2 and 3

Ans) b
Exp) Option b is the correct answer.
A recent study has for the first time tracked the longest in-country migration route of lesser Floricans
from Rajasthan to Maharashtra’s Ahmednagar district.
Statement 1 is incorrect. The lesser florican (Sypheotides indicus), also known as the likh or kharmore
is endemic to the Indian Subcontinent only. They are not found in African continent. The bird breeds
mainly in the central and western parts of India. It is found in India throughout the year and not just in
winters.
Lesser florican is the smallest of India’s three resident bustards (the other two are Bengal florican and
great Indian bustard). It occurs in dry (tall) grasslands with scattered bushes, scrub, and in agricultural
fields of millet, cotton, and some cereal crops.
Statement 2 is correct. The lesser florican has separate breeding and wintering sites with rainfall and land-
use intrinsically linked to the sprightly florican’s breeding habits. Sufficient grass cover is particularly
important during the breeding season. Large trees with big canopy are not desirable in the florican
habitat.
Statement 3 is correct. The lesser florican was included as one of the priority species for recovery by the
Ministry of Environment, Forest and Climate Change (MoEFCC) in 2009, under the scheme of Integrated
Development of Wildlife Habitat.
Knowledge Base:
Integrated Development of Wildlife Habitats’ (IDWH): It is an on-going Centrally Sponsored Scheme in
which financial assistance is provided to State/UT Governments for protection and conservation of wildlife
and its habitats in Protected Areas (PAs) as well as outside PAs and also for the recovery programmes of the
critically endangered species.
Components of the scheme:
1) Support to Protected Areas (National Parks, Wildlife Sanctuaries, Conservation Reserves and
Community Reserves)
2) Protection of Wildlife Outside Protected Areas
3) Recovery programmes for saving critically endangered species and habitats
Source: Longest in-country migration route of lesser florican from Rajasthan tracked-ForumIAS Blog
Lesser florican on verge of extinction in Madhya Pradesh (mongabay.com)
Integrated Development of Wildlife Habitats — Vikaspedia

Forum Learning Centre: Delhi - 2nd Floor, IAPL House, 19 Pusa Road, Karol Bagh, New Delhi - 110005 | Patna - 2nd floor, AG Palace, E Boring Canal Road,
Patna, Bihar 800001 | Hyderabad - 1st & 2nd Floor, SM Plaza, RTC X Rd, Indira Park Road, Jawahar Nagar, Hyderabad, Telangana 500020
9821711605 | https://academy.forumias.com | admissions@forumias.academy | helpdesk@forumias.academy
Page 5 of 38

SFG 2023 | LEVEL 1 | Test #5 – Solutions |


Q.6) Consider the following statements with reference to the Public Accounts Committee:
1. The committee has the equal number of members from both Lok Sabha and Rajya Sabha.
2. The committee examines the annual audit reports of the Comptroller and Auditor General of India.
3. Its recommendations are binding on the ministers.
4. Public account committee is concerned with the questions of policy in broader sense.
Which of the statements given above is/are correct?
a) 2 only
b) 1 and 3 only
c) 2 and 3 only
d) 2 and 4 only

Ans) a
Exp) Option a is the correct answer.
Statement 1 is incorrect: Public account committee was set up first in 1921 under the provisions of the
Government of India Act of 1919 and has since been in existence. At present public account committee
consists of 22 members (15 from the Lok Sabha and 7 from the Rajya Sabha). The members of public
account committee are elected by the Parliament every year from amongst its members according to the
principle of proportional representation by means of the single transferable vote. Thus, all parties get due
representation in it.
Statement 2 is correct: The function of public account committee is to examine the annual audit reports
of the Comptroller and Auditor General of India (CAG), which are laid before the Parliament by the
President. Audit report on appropriation accounts, audit report on finance accounts and audit report on
public undertakings are the three audit reports submitted by the Comptroller and Auditor General of India
(CAG) to the president.
Statement 3 and 4 are incorrect: Public account committees' effectiveness is limited by the following:
1) It is not concerned with the questions of policy in broader sense.
2) It conducts a post-mortem examination of accounts (showing the expenditure already incurred).
3) It cannot intervene in the matters of day-to-day administration
4) Its recommendations are advisory and not binding on the ministries.
5) It is not vested with the power of disallowance of expenditures by the departments.
Source: Indian polity by M Laxmikanth pdf 6th edition. Chapter name-Parliamentary committees. Page no-
580 and 581.

Q.7) Which of the following are among the participants to elect the members to the ‘Legislative Council’ of
a State?
1. Members of local bodies in the State.
2. All graduates of three years standing and residing within the State.
3. Members of the Legislative Assembly of the State.
4. Members of Lok Sabha elected from the State.
5. Members of Rajya Sabha nominated from the State.
Select the correct answer using the code given below:
a) 1, 2 and 3 only
b) 2, 4 and 5 only
c) 1, 2, 4 and 5 only
d) 1, 3, 4 and 5 only

Ans) a
Exp) Option a is the correct answer.

Forum Learning Centre: Delhi - 2nd Floor, IAPL House, 19 Pusa Road, Karol Bagh, New Delhi - 110005 | Patna - 2nd floor, AG Palace, E Boring Canal Road,
Patna, Bihar 800001 | Hyderabad - 1st & 2nd Floor, SM Plaza, RTC X Rd, Indira Park Road, Jawahar Nagar, Hyderabad, Telangana 500020
9821711605 | https://academy.forumias.com | admissions@forumias.academy | helpdesk@forumias.academy
Page 6 of 38

SFG 2023 | LEVEL 1 | Test #5 – Solutions |


There is no uniformity in the organization of State legislatures. Most of the States have a unicameral system,
while others have a bicameral system. At present, only six States have two Houses (bicameral). These are
Andhra Pradesh, Telangana, Uttar Pradesh, Bihar, Maharashtra and Karnataka.
In the States having bicameral system, the State legislature consists of the Governor, the Legislative Council
and the Legislative Assembly. The Legislative Council (Vidhan Parishad) is the upper house (second chamber
or house of elders), while the Legislative Assembly (Vidhan Sabha) is the lower house (first chamber or
popular house).
Unlike the members of the Legislative Assembly, the members of the Legislative Council are indirectly
elected. The maximum strength of the council is fixed at one-third of the total strength of the assembly
and the minimum strength is fixed at 40.
Manner of Election
Of the total number of members of a legislative council:
1) 1/3 are elected by the members of local bodies in the State like municipalities, district boards, etc.,
(Hence Option 1 is correct)
2) 1/12 are elected by graduates of three years standing and residing within the State, (Hence Option 2 is
correct)
3) 1/12 are elected by teachers of three years standing in the State, not lower in standard than secondary
school,
4) 1/3 are elected by the members of the Legislative Assembly of the State from amongst persons who are
not members of the assembly, and (Hence Option 3 is correct)
5) the remainder are nominated by the Governor from amongst persons who have a special knowledge or
practical experience of literature, science, art, cooperative movement and social service.
Members of Lok Sabha elected from the State and Members of Rajya Sabha nominated from the State do
not participate in the elections of the State Legislative Council. (Hence Options 4 and 5 are incorrect)
Thus, 5/6 of the total number of members of a Legislative Council are indirectly elected and 1/6 are
nominated by the Governor. The members are elected in accordance with the system of proportional
representation by means of a single transferable vote.
Source: Pages 711-715, M. Laxmikant 6th Edition.pdf

Q.8) Which of the following statements is/are correct regarding the differences/similarities between the
Presidential form of Government and the Parliamentary form of Government?
1. The Executive is responsible to the legislature in only Parliamentary form of government and not in
Presidential form of government.
2. Unlike in Parliamentary system, President cannot dissolve the lower house under Presidential system.
3. In both Parliamentary system and Presidential system, the governments can be removed through a vote
of no-confidence.
Select the correct answer using the codes given below:
a) 1 and 2 only
b) 1 only
c) 2 and 3 only
d) 1, 2 and 3

Ans) a
Exp) Option a is the correct answer.
A Presidential system is also called a congressional system. It refers to a system of governance in which
the President is the Chief Executive and is elected directly by the people. Whereas, A Parliamentary form
of democracy refers to a system of governance in which the citizens elect representatives to the legislative
Parliament.

Forum Learning Centre: Delhi - 2nd Floor, IAPL House, 19 Pusa Road, Karol Bagh, New Delhi - 110005 | Patna - 2nd floor, AG Palace, E Boring Canal Road,
Patna, Bihar 800001 | Hyderabad - 1st & 2nd Floor, SM Plaza, RTC X Rd, Indira Park Road, Jawahar Nagar, Hyderabad, Telangana 500020
9821711605 | https://academy.forumias.com | admissions@forumias.academy | helpdesk@forumias.academy
Page 7 of 38

SFG 2023 | LEVEL 1 | Test #5 – Solutions |


Statement 1 is correct: The executive is accountable to the legislature only in parliamentary form of
government. There is concentration and fusion of powers between the Legislative and the Executive.
Whereas, in presidential form of government, the executive is not accountable to the legislature. Powers
are separated and the Legislature, the Executive and the Judiciary work separately.
Statement 2 is correct: In parliamentary form of government, the president can dissolve the lower house
and there is dual executive as leader of the state and leader of the government are different. But in
presidential form of government, the President cannot dissolve the lower house. There is a single
executive, as the leader of the state and the leader of the government is the same.
Statement 3 is incorrect: President in Parliamentary form has a fixed tenure and cannot be removed by
a vote of no-confidence. In a Parliamentary form of government, the tenure of the legislature and the
executive are not fixed. If no confidence motion is passed in the house the government will be replaced
with a new government. As the Prime Minister’s tenure depends upon the majority support in the House,
whenever a government fails to prove its majority, the Prime Minister is in no other position but to resign
and the entire Council of Ministers have to step down from power along with him.
While in a Presidential form of government the tenures of both the Legislature and the Executive are fixed.
Neither of them can be removed before the end of their tenure. Elections are held regularly and cannot be
disturbed by passing of no-confidence motion or other procedures.
Source: M laxmikant (Ch 22- Parliament)
Difference between Presidential and Parliamentary form of Government (ipleaders.in)
Difference Between Parliamentary and Presidential Form of Government - GeeksforGeeks

Q.9) With reference to powers of Lok Sabha and Rajya Sabha, consider the following statements:
1. Both Lok Sabha and Rajya Sabha exercises control over executive by introducing No-confidence motion.
2. Under Indian Constitution, Rajya Sabha has no powers with regards to discontinuance of resolution of
national emergency.
3. Lok Sabha can reject all or any of the recommendations of the Rajya Sabha regarding the Money Bill.
Which of the statements given above is/are correct?
a) 1 only
b) 1 and 3 only
c) 2 only
d) 2 and 3 only

Ans) d
Exp) Option d is the correct answer.
Under the Constitution, the Parliament of India consists of three parts viz, the President, the Council of
States and the House of the People. In 1954, the Hindi names ‘Rajya Sabha’ and ‘Lok Sabha’ were adopted by
the Council of States and the House of People respectively. The Rajya Sabha is the Upper House (Second
Chamber or House of Elders) and the Lok Sabha is the Lower House (First Chamber or Popular House). The
former represents the states and union territories of the Indian Union, while the latter represents the
people of India as a whole.
Statement 1 is incorrect: Article 75 of the Constitution says that the council of ministers shall be collectively
responsible to the Lok Sabha. It means that the ministry stays in office so long as it enjoys confidence of
the majority of the members of the Lok Sabha. In other words, the Lok Sabha, not Rajya Sabha can remove
the ministry from office by passing a no-confidence motion.
No confidence motion can be introduced only in the lower house i.e., Lok Sabha. Rajya Sabha is not
permitted to introduce no confidence motion against the Council of Ministers.
Statement 2 is correct: Under Article 352, the President can declare a national emergency when the
security of India or a part of it is threatened by war or external aggression or armed rebellion.

Forum Learning Centre: Delhi - 2nd Floor, IAPL House, 19 Pusa Road, Karol Bagh, New Delhi - 110005 | Patna - 2nd floor, AG Palace, E Boring Canal Road,
Patna, Bihar 800001 | Hyderabad - 1st & 2nd Floor, SM Plaza, RTC X Rd, Indira Park Road, Jawahar Nagar, Hyderabad, Telangana 500020
9821711605 | https://academy.forumias.com | admissions@forumias.academy | helpdesk@forumias.academy
Page 8 of 38

SFG 2023 | LEVEL 1 | Test #5 – Solutions |


A resolution for discontinuance of the national emergency can be passed only in the Lok Sabha. Rajya
Sabha has no powers with regard to discontinuance of National Emergency. The 44th Amendment Act of
1978 provided that, where one-tenth of the total number of members of the Lok Sabha give a written notice
to the Speaker (or to the President if the House is not in session), a special sitting of the House should be
held within 14 days for the purpose of considering a resolution disapproving the continuation of the
proclamation.
Statement 3 is correct: As per the provisions of article 109 of the Constitution, the Rajya Sabha has limited
powers with respect to Money Bills. A Money Bill after having been passed by the Lok Sabha, and sent to
Rajya Sabha for its recommendations, has to be returned to Lok Sabha by the Rajya Sabha, with in a
period of fourteen days from the date of its receipt, with or without recommendations. It is open for the
Lok Sabha, to either accept or reject all or any of the recommendations of the Rajya Sabha.

Difference of powers between Lok Sabha and Rajya Sabha


Source: NCERT Class XI: Indian Constitution at work (Chapter 5: Legislature)
M laxmikant (Chapter 22- Parliament)

Q.10) With reference to ‘Global Gateway Plan’, consider the following statements:
1. The plan aims to invest globally in infrastructure, digital and climate projects.
2. Global Gateway projects will be developed and delivered through Team Europe Initiatives under the
European Commission.
Which of the statements given above is/are correct?
a) 1 only
b) 2 only
c) Both 1 and 2
d) Neither 1 nor 2

Ans) c
Exp) Option c is the correct answer.
The European Commission has announced an international infrastructure plan called the “Global Gateway
Plan”.
Statement 1 is correct. Global Gateway Plan aims to invest €300 billion ($340 billion) globally in
infrastructure, digital and climate projects by 2027. This will help strengthen health, education and

Forum Learning Centre: Delhi - 2nd Floor, IAPL House, 19 Pusa Road, Karol Bagh, New Delhi - 110005 | Patna - 2nd floor, AG Palace, E Boring Canal Road,
Patna, Bihar 800001 | Hyderabad - 1st & 2nd Floor, SM Plaza, RTC X Rd, Indira Park Road, Jawahar Nagar, Hyderabad, Telangana 500020
9821711605 | https://academy.forumias.com | admissions@forumias.academy | helpdesk@forumias.academy
Page 9 of 38

SFG 2023 | LEVEL 1 | Test #5 – Solutions |


research systems across the world. The investment will be made in projects that can be delivered with high
standards, good governance, transparency while ensuring financial sustainability at the same time.
Statement 2 is correct. The Plan will be implemented in a Team Europe approach that brings together
funding by the EU, its Member States and European financial institutions. The decision has been taken by
the European Commission's new European strategy to boost smart, clean and secure links in digital, energy
and transport, as per the press release.
Source: EU plans €300 billion infrastructure fund to counter China’s BRI -ForumIAS Blog
EU launches €300 billion 'Global Gateway' plan to rival China in boosting global infra (republicworld.com)

Q.11) Consider the following statements:


1. In India, there is no law restricting the candidates from contesting in Lok Sabha election from three
constituencies at a time.
2. In 1991 Lok Sabha Election, Shri Devi Lal contested from three Lok Sabha constituencies.
3. As per the existing rules, if a candidate contests in one Lok Sabha election from many constituencies,
his/her party should bear the cost of bye-elections to the constituencies vacated by him/her in the event
of his/her winning in all the constituencies.
Which of the statements given above is/are correct?
a) 1 only
b) 2 only
c) 1 and 3 only
d) 2 and 3 only

Ans) b
Exp) Option b is the correct answer.
Statement 1 is incorrect. In 1996, the Representation of the People Act, 1951, was amended which
restricted the number of seats one candidate could contest in one election to two constituencies. And
whenever they have won in more than one, the candidates can retain only one, forcing by polls in the rest.
Statement 2 is correct. In 1991 Lok Sabha Election, Shri Devi Lal contested from three Lok Sabha
constituencies. (Although such information need not be remembered by students, it can be validated by
eliminating other statements).
Statement 3 is incorrect. There is no such provision that if a candidate contests in one Lok Sabha elections
from many constituencies, then his/her party should bear the cost of bye-elections to the constituencies
vacated by him/her in the event of him/her winning in all the constituencies. In every case, the Election
Commission of India (ECI) bears the cost of elections/bye-elections in all the constituencies in Lok Sabha
and assembly elections as well.
Source) https://economictimes.indiatimes.com/news/politics-and-nation/bar-people-from-
contesting-from-two-seats-election-commission/articleshow/55960421.cms
UPSC CSE 2021

Q.12) With reference to ‘Money bills’, consider the following statements:


1. They can be introduced either by a minister or by a private member.
2. They requires the certification of the speaker whenever transmitted to the Rajya Sabha.
3. Defeat of Money bills introduced in Lok Sabha leads to the resignation of the government.
Which of the statements given above is/are correct?
a) 1 only
b) 1 and 2 only
c) 3 only
d) 2 and 3 only

Forum Learning Centre: Delhi - 2nd Floor, IAPL House, 19 Pusa Road, Karol Bagh, New Delhi - 110005 | Patna - 2nd floor, AG Palace, E Boring Canal Road,
Patna, Bihar 800001 | Hyderabad - 1st & 2nd Floor, SM Plaza, RTC X Rd, Indira Park Road, Jawahar Nagar, Hyderabad, Telangana 500020
9821711605 | https://academy.forumias.com | admissions@forumias.academy | helpdesk@forumias.academy
Page 10 of 38

SFG 2023 | LEVEL 1 | Test #5 – Solutions |


Ans) d
Exp) Option d is the correct answer.
Article 110 of the Constitution deals with the definition of Money bills. Money bills are concerned with
financial matters like taxation, public expenditure. Whereas, the bills mentioned in Articles 117 (1) and
Article 117 (3) are financial bills.
Statement 1 is incorrect: Ordinary bills can be introduced either by a minister or by a private member.
Whereas, Money bill can be introduced only by a minister. It cannot be introduced by any private member.
Statement 2 is correct: Money bills always requires the certification of the speaker whenever transmitted
to the Rajya Sabha. Whereas, ordinary bills do not require the certification of the speaker when
transmitted to the Rajya Sabha (if it has originated in the Lok Sabha).
Statement 3 is correct: Money bill’s defeat in the Lok Sabha leads to the resignation of the government.
Defeat of non-Money bills introduced by a minister in Lok Sabha may lead to the resignation of the
government. But if ordinary or non-money bill is introduced by any private member of the parliament then
defeat will not lead to the resignation of the government.
Source: M Laxmikanth (Chapter- 22, Parliament)

Q.13) What are the ways through which Parliament controls the executive?
1. Not passing Motion of Thanks
2. Passing of cut motion
3. Anti-defection law
4. Half-an-hour discussion
5. Removal of Deputy-speaker of Lok Sabha
Select the correct answer using the code given below:
a) 1 and 3 only
b) 1, 2 and 4 only
c) 2, 3 and 4 only
d) 1, 3 and 5 only

Ans) b
Exp) Option b is the correct answer.
The Constitution of India established a parliamentary form of government in which the Executive is
responsible to the Parliament for its policies and acts. The Parliament exercises control over the
Executive through question-hour, zero hour, half-an-hour discussion, short duration discussion, calling
attention motion, adjournment motion, no-confidence motion, censure motion and other discussions.
It also supervises the activities of the Executive with the help of its committees like committee on
government assurance, committee on subordinate legislation, committee on petitions, etc
Statement 1 is correct: The first session after each general election and the first session of every fiscal year
is addressed by the President. In this address, the President outlines the policies and programmes of the
government in the preceding year and ensuing year. This address of the President, is discussed in both the
Houses of Parliament on a motion called the ‘Motion of Thanks’. At the end of the discussion, the motion is
put to vote. This motion must be passed in the House. Otherwise, it amounts to the defeat of the
government. This inaugural speech of the President is an occasion available to the members of Parliament
to raise discussions and debates to examine and criticise the government and administration for its lapses
and failures.
Statement 2 is correct: The ministers are collectively responsible to the Parliament in general and to the
Lok Sabha in particular. The Lok Sabha can also express lack of confidence in the government by passing

Forum Learning Centre: Delhi - 2nd Floor, IAPL House, 19 Pusa Road, Karol Bagh, New Delhi - 110005 | Patna - 2nd floor, AG Palace, E Boring Canal Road,
Patna, Bihar 800001 | Hyderabad - 1st & 2nd Floor, SM Plaza, RTC X Rd, Indira Park Road, Jawahar Nagar, Hyderabad, Telangana 500020
9821711605 | https://academy.forumias.com | admissions@forumias.academy | helpdesk@forumias.academy
Page 11 of 38

SFG 2023 | LEVEL 1 | Test #5 – Solutions |


a Cut Motion. It is a power given to the members of the Lok Sabha that enables its members to oppose any
demand in a Finance Bill proposed by a government.
Statement 3 is incorrect: The anti-defection law punishes individual Members of Parliament (MPs) for
leaving one party for another. It is intended to regulate the defecting MPs. It does not regulate or control
executive in any manner.
Statement 4 is correct: Through Half-an-Hour Discussion, Parliament can discuss a matter of sufficient
public importance, which has been subjected to a lot of debate and the answer to which needs elucidation
on a matter of fact. This keeps executive accountable to the Parliament.
Statement 5 is incorrect: Deputy Speaker is not the part of the executive and hence, his/her removal
does not impact Parliaments control over executive. The position of Deputy Speaker is usually offered to
opposition party in India and he acts as a chairperson in the public accounts committee which functions
as checks on the executive.
Source: M Laxmikanth (Chapter-22, Parliament)
Anti-defection annual conference (prsindia.org)

Q.14) In which of the following cases, a member of Parliament will be liable to be disqualified on the ground
of defection according to the Tenth Schedule?
1. If he/she voluntarily gives up membership of his party after being elected as Presiding officer of house.
2. If he/she votes contrary to any direction issued by his political party.
3. If he/she, as a nominated member, joins any political party within six months of taking his seat.
4. If he/she, as an independent member joins any political party after being elected.
Select the correct answer using the code given below:
a) 1 and 3 only
b) 2 only
c) 2 and 4 only
d) 2, 3 and 4 only

Ans) c
Exp) Option c is the correct answer.
The Tenth Schedule contains the provisions with respect to the disqualification of members of Parliament
and the state legislatures on the ground of defection. It has been made to stop the horse trading in the
politics and increase the faith in the democracy.
Statement 1 is incorrect: The disqualification on the ground of defection does not apply if a member, after
being elected as the presiding officer of the House, voluntarily gives up the membership of his party. This
exemption has been provided in view of the dignity and impartiality of this office.
Statement 2 is correct: A member of a House belonging to any political party becomes disqualified for
being a member of the House, if he voluntarily gives up his membership of such political party or he votes
or abstains from voting in such House contrary to any direction issued by his political party without
obtaining prior permission of such party.
Statement 3 is incorrect: If a nominated members joins any political party within six months of taking his
seat, he will not be considered as a case of defection. A nominated member of a House becomes disqualified
for being a member of the House if he joins any political party after the expiry of six months from the date
on which he takes his seat in the House.
Statement 4 is correct: An independent member of a House (elected without being set up as a candidate
by any political party) becomes disqualified to remain a member of the House if he joins any political party
after such election. There is no any provision of getting approval from president before joining any party
to escape from defection.
Source: M laxmikant (Chapter 76, Anti defection law)

Forum Learning Centre: Delhi - 2nd Floor, IAPL House, 19 Pusa Road, Karol Bagh, New Delhi - 110005 | Patna - 2nd floor, AG Palace, E Boring Canal Road,
Patna, Bihar 800001 | Hyderabad - 1st & 2nd Floor, SM Plaza, RTC X Rd, Indira Park Road, Jawahar Nagar, Hyderabad, Telangana 500020
9821711605 | https://academy.forumias.com | admissions@forumias.academy | helpdesk@forumias.academy
Page 12 of 38

SFG 2023 | LEVEL 1 | Test #5 – Solutions |


Q.15) Consider the following statements regarding ‘She is a Changemaker’ Programme:
1. It is a pan-India scheme that aims to impart vocational training to women in various technical crafts.
2. It has been launched by the National Commission for Women.
Which of the statements given above is/are correct?
a) 1 only
b) 2 only
c) Both 1 and 2
d) Neither 1 nor 2

Ans) b
Exp) Option b is the correct answer.
Statement 1 is incorrect. The ‘She is a Changemaker Programme’ programme aims to undertake capacity
building of women political leaders and improve their decision-making and communication skills
including, oration, writing etc. The programme is for women representatives at all levels, gram panchayats
to parliament members and political workers including office bearers of National/State political parties.
The programme will benefit every woman who wishes to make an identity for herself in politics and help
her to claim her rightful space in politics.
Statement 2 is correct. National Commission for Women (NCW) has launched a pan-India capacity
building programme named ‘She is a Changemaker’. The programme will be undertaken by NCW in
association with region-wise training institutes.
Source: NCW Launches a Pan-India Capacity Building Programme ‘She is a Changemaker’ for Women in
Politics-ForumIAS Blog

Q.16) A Joint Session of Parliament can be called for which of the following types of bills?
1. Money bill
2. Ordinary bill
3. Financial bills (I)
4. Constitutional Amendment bills under Article 368
Select the correct answer using the code given below:
a) 1, 2 and 3 only
b) 2, 3 and 4 only
c) 2 and 3 only
d) 2 only

Ans) c
Exp) Option c is the correct answer.
Joint sitting is an extraordinary machinery provided by the Constitution aimed to maintain a much-
needed synergy between the two houses of the Parliament. Article 118 provides that President of India
may after consultation with the chair of the Rajya Sabha and the Speaker of Lok Sabha may make rules for
procedure of joint session of parliament.
Statements 2 and 3 are correct:
1) Ordinary Bill and Financial bills (I) are sent for the President’s assent only after being approved by both
the Houses. In case of a deadlock due to disagreement between the two Houses, a joint sitting of both
the houses can be summoned by the president to resolve the deadlock.
Statements 1 and 4 are incorrect:
1) Money bill is sent for the President’s assent even if it is approved by only Lok Sabha. There is no chance
of any disagreement between the two Houses as the Rajya Sabha has limited rights over money bills.
Hence, there is no provision of joint sitting of both the Houses in this regard.

Forum Learning Centre: Delhi - 2nd Floor, IAPL House, 19 Pusa Road, Karol Bagh, New Delhi - 110005 | Patna - 2nd floor, AG Palace, E Boring Canal Road,
Patna, Bihar 800001 | Hyderabad - 1st & 2nd Floor, SM Plaza, RTC X Rd, Indira Park Road, Jawahar Nagar, Hyderabad, Telangana 500020
9821711605 | https://academy.forumias.com | admissions@forumias.academy | helpdesk@forumias.academy
Page 13 of 38

SFG 2023 | LEVEL 1 | Test #5 – Solutions |


2) Constitution Amendment Bill: As per Article 368, the Indian Constitution can be amended by both
houses of parliament by 2/3rd majority. In case of disagreement between both houses, there is no
provision to summon joint session of parliament.
Knowledge Base:
When can Joint Sitting be called?
A Joint Sitting can be called under any one of the following three situations after a bill has been passed by
one House and transmitted to the other House:
1. if the bill is rejected by the other House;
2. if the Houses have finally disagreed as to the amendments to be made in the bill; or
3. if more than six months have elapsed from the date of the receipt of the bill by the other House without
the bill being passed by it.
In the above three situations, the president can summon both the Houses to meet in a joint sitting for the
purpose of deliberating and voting on the bill.
Source: M laxmikant (Chapter 22, Parliament)
[Answered] joint session of the two houses of the Parliament. -ForumIAS Blog

Q.17) With reference to Rajya Sabha, consider the following statements:


1. Rajya Sabha can authorise the Parliament to create new All-India Services.
2. A resolution for the removal of the Vice-President can be introduced only in the Rajya Sabha.
Which of the statements given above is/are correct?
a) 1 only
b) 2 only
c) Both 1 and 2
d) Neither 1 nor 2

Ans) c
Exp) Option c is the correct answer.
The Rajya Sabha is the upper house of the bicameral Parliament of India. As of 2021 it has a maximum
membership of 245, of which 233 are elected by the legislatures of the states and union territories using
single transferable votes through open ballots, while the president can appoint 12 members for their
contributions to art, literature, science, and social services.
Statement 1 is correct: Under Article 312 of Indian Constitution, Rajya Sabha can authorise the Parliament
to create new All-India Services common to both the Centre and the states.
Statement 2 is correct: Rajya Sabha alone can initiate a move for the removal of the vice-president. In
other words, a resolution for the removal of the vice-president can be introduced only in the Rajya Sabha
and not in the Lok Sabha (Article 67).
Source: M Laxmikanth (Ch 22: Parliament)
NCERT Class XI: Indian Constitution at work (Chapter 5: Legislature)

Q.18) With reference to Parliamentary Elections after Independence, consider the following statements:
1. The highest number of seats ever in any election to Lok Sabha was won by the Janta party in 1984.
2. Seventeenth Lok Sabha of 2019 has the highest percentage of women Member of Parliaments since
Independence.
3. The 10th Lok Sabha election of 1991 recorded the highest voter participations since independence.
4. The first ever coalition government in Centre was formed by Communist Party of India (M) in 1977.

Forum Learning Centre: Delhi - 2nd Floor, IAPL House, 19 Pusa Road, Karol Bagh, New Delhi - 110005 | Patna - 2nd floor, AG Palace, E Boring Canal Road,
Patna, Bihar 800001 | Hyderabad - 1st & 2nd Floor, SM Plaza, RTC X Rd, Indira Park Road, Jawahar Nagar, Hyderabad, Telangana 500020
9821711605 | https://academy.forumias.com | admissions@forumias.academy | helpdesk@forumias.academy
Page 14 of 38

SFG 2023 | LEVEL 1 | Test #5 – Solutions |


Which of the statement given above is/are correct?
a) 1 and 2 only
b) 2 only
c) 1, 2 and 4 only
d) 3 and 4 only

Ans) b
Exp) Option b is the correct answer.
Statement 1 is incorrect: Congress party under the leadership of Rajiv Gandhi won the highest seat ever
(Congress 415) in the general election of Eighth Lok Sabha (1984). Riding on the wave of sympathy in
response to assassination of Indira Gandhi, the Congress party under Rajiv Gandhi’s leadership (son of
Indira Gandhi) came to power in a landslide victory.
Statement 2 is correct: In the general election of seventeenth Lok Sabha (2019) election, out of the 716
women candidates who contested elections, 78 have been elected, which is 14% of the house. This is higher
than sixteenth Lok Sabha election (2014) where there were 62-woman MPs in the house and the highest
ever since independence.
Statement 3 is incorrect: The general election of 17th Lok Sabha (2019) elections registered a record voter
turnout of 67.11%, highest since the independence. This record breached the previous turnout of 65.95%
voting notched up in 2014, according to tentative data released by the Election Commission (EC). Voting
was the lowest ever in parliamentary elections of 1991 with just 53 per cent of the electorate exercising
their right to vote.
Statement 4 is incorrect: A coalition government is a form of government in which political parties
cooperate to form a government. Janta party (not the Communist Party of India (M)) formed the first
ever coalition government in Centre under the leadership of Morarji Desai (1977-1977). But the first
coalition government to complete its full-term was the Atal Bihari Vajpayee-led National Democratic
Alliance from 1999.
Source: M Laxmikanth (Chapter 71, Elections; Chapter 75, Coalition government)
History of Indian Parliament Elections (Lok Sabha)- Factly

Q.19) Arrange the following items in the correct order of the passing of an ordinary bill in the Parliament:
1. Consideration Stage
2. Stage of General Discussion
3. Third Reading
4. Assent of the President
5. Publication of the bill in the Gazette
Select the correct answer from the code given below:
a) 1-2-3-4-5
b) 2-1-3-4-5
c) 5-2-1-3-4
d) 5-1-2-3-4

Ans) c
Exp) Option c is the correct answer.
A bill is a proposal for legislation and it becomes an act or law when duly enacted. Every ordinary bill has
to pass through the following five stages in the Parliament before it finds a place on the Statute Book:
1) First Reading - The introduction of the bill and its publication in the Gazette constitute the first reading
of the bill. No discussion on the bill takes place at this stage.

Forum Learning Centre: Delhi - 2nd Floor, IAPL House, 19 Pusa Road, Karol Bagh, New Delhi - 110005 | Patna - 2nd floor, AG Palace, E Boring Canal Road,
Patna, Bihar 800001 | Hyderabad - 1st & 2nd Floor, SM Plaza, RTC X Rd, Indira Park Road, Jawahar Nagar, Hyderabad, Telangana 500020
9821711605 | https://academy.forumias.com | admissions@forumias.academy | helpdesk@forumias.academy
Page 15 of 38

SFG 2023 | LEVEL 1 | Test #5 – Solutions |


The member who wants to introduce the bill has to ask for the leave of the House. When the House grants
leave to introduce the bill, the mover of the bill introduces it by reading its title and objectives. Later, the
bill is published in the Gazette of India. If a bill is published in the Gazette before its introduction, leave of
the House to introduce the bill is not necessary.
2) Second Reading - During this stage, the bill receives not only the general but also the detailed scrutiny
and assumes its final shape. This stage involves three more sub-stages, namely, stage of general
discussion, committee stage and consideration stage.
(a) Stage of General Discussion - The principles of the bill and its provisions are discussed generally, but
the details of the bill are not discussed.
(b) Committee Stage - This committee examines the bill thoroughly and in detail, clause by clause. It can
also amend its provisions, but without altering the principles underlying it.
(c) Consideration Stage - The House, after receiving the bill from the select committee, considers the
provisions of the bill clause by clause. Each clause is discussed and voted upon separately. The members
can also move amendments and if accepted, they become part of the bill.
3) Third Reading - At this stage, the debate is confined to the acceptance or rejection of the bill as a whole
and no amendments are allowed. If the majority of members present and voting accept the bill, the bill is
regarded as passed by the House and transmitted to the second House.
4) Bill in the Second House - the bill passes through all the three stages, that is, first reading, second reading
and third reading.
5) Assent of the President - Every bill after being passed by both Houses of Parliament either singly or at
a joint sitting, is presented to the president for his assent. If the president gives his assent to the bill, the
bill becomes an act and is placed on the Statute Book.
So, for the given question the correct order would be: Publication of the bill in the Gazette - Stage of
General Discussion - Consideration Stage - Third Reading - Assent of the President.
Source: Indian Polity by Laxmikant – 6th Edition – Chapter 22 Parliament.

Q.20) With reference to ‘Socio-Economic Policies of States’, which one of the following statements best
describes the term ‘New Welfarism”?
a) It is a policy of prioritising the supply of public goods such as basic health and primary education through
top-down approach.
b) It is a form of neo-liberal policy that seeks for greater role of private sector in welfare sectors like
education and health.
c) It focuses on the role of state in providing subsidized tangible essential goods and services such as bank
accounts, cooking gas, toilets, electricity etc.
d) It is an approach that entails direct monetary benefits to the targeted group instead of providing physical
support system.

Ans) c
Exp) Option c is the correct answer.
Recently released National Family Health Survey (NFHS-5) shows remarkable improvement in New
Welfarism.
New Welfarism has entailed the subsidised public provision of essential goods and services, normally
provided by the private sector. Such as bank accounts, cooking gas, toilets, electricity, housing, etc.
New Welfarism does not prioritise the supply of public goods such as basic health and primary education
as governments have done around the world historically.
NFHS-5 data show the success of New Welfarism:
1) Firstly, striking improvements in household access to improved sanitation, cooking gas and bank
accounts used by women.

Forum Learning Centre: Delhi - 2nd Floor, IAPL House, 19 Pusa Road, Karol Bagh, New Delhi - 110005 | Patna - 2nd floor, AG Palace, E Boring Canal Road,
Patna, Bihar 800001 | Hyderabad - 1st & 2nd Floor, SM Plaza, RTC X Rd, Indira Park Road, Jawahar Nagar, Hyderabad, Telangana 500020
9821711605 | https://academy.forumias.com | admissions@forumias.academy | helpdesk@forumias.academy
Page 16 of 38

SFG 2023 | LEVEL 1 | Test #5 – Solutions |


2) Secondly, Improvement in case of stunting and diarrhoea on child-related outcomes.
3) Thirdly, India witnesses a non-monopoly of any political party in regional performance. For instance,
the improvements in Rajasthan have happened under the Congress, in MP and Haryana under the BJP,
in Odisha under the BJD, and in UP under both Samajwadi Party and BJP.
Source: What the latest NFHS data says about the New Welfarism-ForumIAS Blog

Q.21) In India, which of the following reviews the independent regulators in sectors like
telecommunications, insurance, electricity, etc.?
1. Ad Hoc Committee set up by the Parliament.
2. Parliamentary Department Related Standing Committee
3. Finance Commission
4. Financial Sector Legislative Reforms Commission
5. NITI Aayog
Select the correct answer using the code given below.
a) 1 and 2 only
b) 1, 3 and 4 only
c) 3, 4 and 5 only
d) 2 and 5 only

Ans) a
Exp) Option a is the correct answer.
Independent regularity authorities are agencies of modern democratic governments. They are parts of the
executive wing with a certain degree of statutory or constitutional autonomy, reporting directly to the
legislature. Like the general executive, they are accountable to the legislature and subject to judicial review.
Ad Hoc Committee set up by the Parliament and Parliamentary Department Related Standing Committee
review the independent regulators in sectors like telecommunications, insurance electricity, etc.
Finance Commission and NITI Aayog are advisory bodies and do not review the independent regulators in
sectors like telecommunications. Financial Sector Legislative Reforms Commission (FSLRC) also had no role
in reviewing independent regulators.
Source) UPSC CSE 2019

Q.22) With reference to the method of Proportional Representation for elections, consider the following
statements:
1. The system of Territorial Representation represents the electorate more effectively as compared to
Proportional Representation system.
2. The List System of Proportional Representation is adopted for the election of the Rajya Sabha members.
3. This system does not give any scope for organising by-elections.
Which of the statements given above is/are correct?
a) 1 only
b) 3 only
c) 1 and 2 only
d) 2 and 3 only

Ans) b
Exp) Option b is the correct answer.
Proportional representation is the idea that the seats in parliament should be in proportion to the votes
cast.

Forum Learning Centre: Delhi - 2nd Floor, IAPL House, 19 Pusa Road, Karol Bagh, New Delhi - 110005 | Patna - 2nd floor, AG Palace, E Boring Canal Road,
Patna, Bihar 800001 | Hyderabad - 1st & 2nd Floor, SM Plaza, RTC X Rd, Indira Park Road, Jawahar Nagar, Hyderabad, Telangana 500020
9821711605 | https://academy.forumias.com | admissions@forumias.academy | helpdesk@forumias.academy
Page 17 of 38

SFG 2023 | LEVEL 1 | Test #5 – Solutions |


Statement 1 is incorrect: Under territorial representation, every member of the legislature represents a
geographical area known as a constituency. From each constituency, only one representative is elected. In
this system, a candidate who secures majority of votes is declared elected. This simple majority system of
representation does not represent the whole electorate. The system of proportional representation aims
at removing the defects of territorial representation. Under this system, all sections of the people get
representation in proportion to their number.
Statement 2 is incorrect: There are two kinds of proportional representation, namely, single transferable
vote system and list system. In India, the single transferable vote system is adopted for the election of
members to the Rajya Sabha and state legislative council and for electing the President and the Vice-
President.
Single transferable vote is an electoral system in which seats are allocated to candidates in order to achieve
proportional representation, and where each candidate is ranked in order of preference by voters. As the
vote-count develops and candidates are either elected or eliminated, it is transferred to other candidates
in line with the voter’s preferences.
In the list system, electors vote for one of several lists of candidates, usually prepared by the political
parties. Each party is granted seats in proportion to the number of popular votes it receives.
Statement 3 is correct: The system of proportional representation has the following demerits:
1) It is highly expensive.
2) It does not give any scope for organising by-elections.
3) It eliminates intimate contacts between voters and representatives.
4) It promotes minority thinking and group interests.
5) It increases the significance of party system and decreases that of voter.
Source: Indian Polity by Laxmikant – 6th Edition – Chapter 22 Parliament.
https://uk-engage.org/2013/08/what-are-the-advantages-and-disadvantages-of-using-a-
proportional-representation-pr-electoral-system/

Q.23) With reference to ‘Delimitation Commission’ in India, consider the following statements:
1. The orders of the Commission cannot be called in question before any court.
2. Modifications to orders of the Commission can only be done by the Lok Sabha and not by State Legislative
Assemblies.
Which of the statements given above is/are correct?
a) 1 only
b) 2 only
c) Both 1 and 2
d) Neither 1 nor 2

Ans) a
Exp) Option a is the correct answer.
Delimitation literally means the act or process of fixing limits or boundaries of territorial constituencies in
a country or a province having a legislative body. The job of delimitation is assigned to a high-power body.
Such a body is known as Delimitation Commission or a Boundary Commission.
Statement 1 is correct: The Delimitation Commission in India is a high-power body whose orders have
the force of law and cannot be called in question before any court. These orders come into force on a date
to be specified by the President of India in this behalf.
Statement 2 is incorrect: The copies of its orders are laid before the Lok Sabha and the State Legislative
Assembly concerned, but no modifications are permissible therein by them.
Source: https://eci.gov.in/delimitation-website/delimitation/

Forum Learning Centre: Delhi - 2nd Floor, IAPL House, 19 Pusa Road, Karol Bagh, New Delhi - 110005 | Patna - 2nd floor, AG Palace, E Boring Canal Road,
Patna, Bihar 800001 | Hyderabad - 1st & 2nd Floor, SM Plaza, RTC X Rd, Indira Park Road, Jawahar Nagar, Hyderabad, Telangana 500020
9821711605 | https://academy.forumias.com | admissions@forumias.academy | helpdesk@forumias.academy
Page 18 of 38

SFG 2023 | LEVEL 1 | Test #5 – Solutions |


Q.24) In India, President is an integral part of the Parliament. In this context, consider the following
statements:
1. A bill passed by both the houses of Parliament cannot become law without the President’s assent.
2. S/he is the Leader of the House of the Lok Sabha.
3. S/he presides over joint sitting of two Houses of Parliament.
Which of the statements given above is/are correct?
a) 1 only
b) 2 and 3 only
c) 1 and 2 only
d) 1 and 3 only

Ans) a
Exp) Option a is the correct answer.
Under the Indian constitution, the Parliament of India consists of three parts viz, the President, the Council
of states (Rajya Sabha) and the House of the people (Lok Sabha).
Statement 1 is correct: Though the president of India is not a member of either House of Parliament and
does not sit in the Parliament to attend its meetings, still he is an integral part of Parliament. This is because,
a bill passed by both the houses of Parliament cannot become law without the President’s assent.
Statement 2 is incorrect: Leader of the House in Lok Sabha is the Prime Minister by default if she/he is
a member of the Lok Sabha. If the Prime Minister is not a member of Parliament’s Lower House, she or he
might appoint another minister to serve as Leader of the House.
Statement 3 is incorrect: Lok Sabha Speaker presides over a joint sitting of the two Houses of
Parliament. Such a sitting is summoned by the President to settle a deadlock between the two Houses on
a bill.
Source: Indian Polity by Laxmikant – 6th Edition – Chapter 22 Parliament

Q.25) Consider the following statements with reference to Public Key Cryptography:
1. It is a communication where people exchange messages that can only be read by one another.
2. Data that is encrypted with the public key can be decrypted only with the corresponding private key.
Which of the statements given above is/are correct?
a) 1 only
b) 2 only
c) Both 1 and 2
d) Neither 1 nor 2

Ans) c
Exp) Option c is the correct answer.
The research undertaken by the Indian Army in the field of Quantum Technology will help leapfrog into
next-generation communication and transform the current system of cryptography in the Indian Armed
Forces to Post Quantum Cryptography (PQC).
Statement 1 is correct. Public-key cryptography also called asymmetric cryptography is communication
where people exchange messages that can only be read by one another. In the Symmetric-key
Cryptography both the sender and receiver share a single key. The sender uses this key to encrypt plaintext
and send the cipher text to the receiver. On the other side the receiver applies the same key to decrypt the
message and recover the plain text.
Statement 2 is correct. Data that is encrypted with the public key can be decrypted only with the
corresponding private key. In Public-Key Cryptography two related keys (public and private key) are

Forum Learning Centre: Delhi - 2nd Floor, IAPL House, 19 Pusa Road, Karol Bagh, New Delhi - 110005 | Patna - 2nd floor, AG Palace, E Boring Canal Road,
Patna, Bihar 800001 | Hyderabad - 1st & 2nd Floor, SM Plaza, RTC X Rd, Indira Park Road, Jawahar Nagar, Hyderabad, Telangana 500020
9821711605 | https://academy.forumias.com | admissions@forumias.academy | helpdesk@forumias.academy
Page 19 of 38

SFG 2023 | LEVEL 1 | Test #5 – Solutions |


used. Public key may be freely distributed, while its paired private key remains a secret. The public key is
used for encryption and for decryption private key is used.

Source: https://pib.gov.in/PressReleaseIframePage.aspx?PRID=1786012
https://economictimes.indiatimes.com/definition/cryptography

Q.26) With reference to the Legislative Council in a State, consider the following Statements:
1. A Legislative Council can be created by the Parliament only when Rajya Sabha passes a resolution to that
effect.
2. The Act of Parliament to create a Legislative Council is considered a Constitutional Amendment under
Article 368.
3. The actual strength of the Legislative Council in a State is fixed by the Governor of that State.
4. The retiring members of the Council are eligible for re-election and re-nomination for any number of
times.
Which of the Statements given above is/are correct?
a) 2 and 3 only
b) 4 only
c) 3 and 4 only
d) 1, 2, 3 only

Ans) b
Exp) Option b is the correct answer.
There is no uniformity in the organisation of State legislatures. Most of the States have a unicameral system,
while others have a bicameral system. At present, only six States have two Houses (bicameral). These are
Andhra Pradesh, Telangana, Uttar Pradesh, Bihar, Maharashtra and Karnataka.
In the States having bicameral system, the State legislature consists of the Governor, the Legislative Council
and the Legislative Assembly. The Legislative Council (Vidhan Parishad) is the upper house (second chamber
or house of elders), while the Legislative Assembly (Vidhan Sabha) is the lower house (first chamber or
popular house).
Statement 1 is incorrect: The Constitution provides for the abolition or creation of legislative councils in
states. Accordingly, the Parliament can abolish a legislative council (where it already exists) or create it
(where it does not exist), if the legislative assembly of the concerned state passes a resolution to that
effect.
Statement 2 is incorrect: Such a specific resolution must be passed by the state assembly by a special
majority, that is, a majority of the total membership of the assembly and a majority of not less than two-
thirds of the members of the assembly present and voting. This Act of Parliament is not to be deemed as

Forum Learning Centre: Delhi - 2nd Floor, IAPL House, 19 Pusa Road, Karol Bagh, New Delhi - 110005 | Patna - 2nd floor, AG Palace, E Boring Canal Road,
Patna, Bihar 800001 | Hyderabad - 1st & 2nd Floor, SM Plaza, RTC X Rd, Indira Park Road, Jawahar Nagar, Hyderabad, Telangana 500020
9821711605 | https://academy.forumias.com | admissions@forumias.academy | helpdesk@forumias.academy
Page 20 of 38

SFG 2023 | LEVEL 1 | Test #5 – Solutions |


an amendment of the Constitution for the purposes of Article 368 and is passed like an ordinary piece of
legislation (ie, by simple majority).
Statement 3 is incorrect: Unlike the members of the legislative assembly, the members of the legislative
council are indirectly elected. The maximum strength of the council is fixed at one-third of the total
strength of the assembly and the minimum strength is fixed at 40. It means that the size of the council
depends on the size of the assembly of the concerned state. This is done to ensure the predominance of
the directly elected House (assembly) in the legislative affairs of the state. Though the Constitution has
fixed the maximum and the minimum limits, the actual strength of a Council is fixed by Parliament.
Statement 4 is correct: Like the Rajya Sabha, the legislative council is a continuing chamber, that is, it is a
permanent body and is not subject to dissolution. But, one-third of its members retire on the expiration of
every second year. So, a member continues as such for six years. The vacant seats are filled up by fresh
elections and nominations (by governor) at the beginning of every third year. The retiring members are
also eligible for re-election and re-nomination any number of times.
Source: 725-728, M. Laxmikant 6th Edition.pdf

Q.27) With reference to the Anglo-Indians, consider the following statements:


1. The term ‘Anglo-Indian’ is defined in the Indian Constitution
2. The original Indian Constitution of 1950 provided for the reservation for Anglo-Indians in Lok Sabha.
3. The 104th Constitutional Amendment Act, 2020 discontinued the reservation of Anglo-Indians in the Lok
Sabha.
Which of the statements given above are correct?
a) 1 and 2 only
b) 2 and 3 only
c) 1 and 3 only
d) 1, 2 and 3

Ans) d
Exp) Option d is the correct answer.
The Constitution provides for the reservation of seats for certain classes of citizens like the SCs and STs.
Statement 1 is correct. The term Anglo-Indians is defined by the Constitution. Article 366 defines Anglo-
Indian as a person whose father or any of whose other male progenitors in the male line is or was of
European descent but who is domiciled within the territory of India and is or was born within such territory
of parents habitually resident therein and not established there for temporary purposes only.
Statement 2 is correct: The provision for reservation for Anglo-Indian in Lok Sabha was provided in the
original Indian Constitution. Originally, this provision was to operate till 1960 but has been extended by
the Eighth Amendment Act, 1960 for a period of ten years (i.e., up to 1970). It was extended till 2020 by the
95th Amendment Act, 2009.
Statement 3 is correct: The 104th Constitutional Amendment Act. 2020 extend the reservation of seats for
SCs and STs in the Lok Sabha and states assemblies from Seventy years to Eighty years but did not extend
the reserved seats for the Anglo-Indian community in the Lok Sabha and state assemblies.
Knowledge Base:
Article 366(2) of the original Constitution defines the Anglo-Indian as a person whose father or any other
male progenitor in the male line is or was of European descent, but who is domiciled within the territory
of India and is or was born within such territory of parents habitually resident therein and not established
there for temporary purposes only.
Source: Indian Polity by Laxmikant – 6th Edition – Chapter 22 Parliament.

Forum Learning Centre: Delhi - 2nd Floor, IAPL House, 19 Pusa Road, Karol Bagh, New Delhi - 110005 | Patna - 2nd floor, AG Palace, E Boring Canal Road,
Patna, Bihar 800001 | Hyderabad - 1st & 2nd Floor, SM Plaza, RTC X Rd, Indira Park Road, Jawahar Nagar, Hyderabad, Telangana 500020
9821711605 | https://academy.forumias.com | admissions@forumias.academy | helpdesk@forumias.academy
Page 21 of 38

SFG 2023 | LEVEL 1 | Test #5 – Solutions |


Q.28) Which of the following States/Union Territories in India have bicameral legislature?
1. Puducherry
2. Andhra Pradesh
3. Uttar Pradesh
4. Rajasthan
5. Bihar
6. Karnataka
7. Jammu & Kashmir
Select the correct answer using the code given below:
a) 1, 2, 3, 5 and 7 only
b) 2, 3, 5 and 6 only
c) 2, 3, 4, 5 and 6 only
d) 1, 2, 3, 5 and 6 only

Ans) b
Exp) Option b is the correct answer.
Bicameralism is the practice of having two Houses of Parliament. At the State level, the equivalent of the
Lok Sabha is the Vidhan Sabha (Legislative Assembly), and that of the Rajya Sabha is the Vidhan Parishad
(Legislative Council). Under Article 169, Parliament may by law create or abolish the second chamber in a
State if the Legislative Assembly of that State passes a resolution to that effect by a special majority. At
present, six Indian States have bicameral legislatures.
Pondicherry and Rajasthan do not have a bi-cameral legislature (Hence, option 1 and 4 are incorrect)
Only Six States in India are having a Bicameral Legislature ie Both Legislative Assembly and Legislative
Council:
1) Andhra Pradesh (Hence, option 2 is correct)
2) Telangana
3) Uttar Pradesh (Hence, option 3 is correct)
4) Bihar (Hence, option 5 is correct)
5) Maharashtra
6) Karnataka. (Hence, option 6 is correct)
In 2019, the Jammu & Kashmir Legislative Council was abolished through the J&K Reorganisation Act,
2019. This act reduced the State of J&K to the Union Territories of J&K and Ladakh. (Hence, option 7 is
incorrect)
Note: In 2020, Andhra Pradesh Legislative Assembly passed the resolution for abolition of the Legislative
Council. This resolution is yet to be cleared by the Parliament of India to finally abolish the council.
Knowledge Base:
Article 169 (Creation and Abolition): The Parliament can abolish a Legislative Council (where it already
exists) or create it (where it does not exist) by a simple majority, that is, a majority of the members of each
House present and voting, if the legislative assembly of the concerned state, by a special majority, passes a
resolution to that effect. Special majority implies a majority of the total membership of the assembly and a
majority of not less than two-thirds of the members of the assembly present and voting.
Source: M. Laxmikant 6th Edition.pdf
https://www.thehindu.com/opinion/op-ed/what-is-bicameral-legislature/article26763394.ece
https://prsindia.org/articles-by-prs-team/bengal-wants-upper-house-back-how-states-have-councils

Forum Learning Centre: Delhi - 2nd Floor, IAPL House, 19 Pusa Road, Karol Bagh, New Delhi - 110005 | Patna - 2nd floor, AG Palace, E Boring Canal Road,
Patna, Bihar 800001 | Hyderabad - 1st & 2nd Floor, SM Plaza, RTC X Rd, Indira Park Road, Jawahar Nagar, Hyderabad, Telangana 500020
9821711605 | https://academy.forumias.com | admissions@forumias.academy | helpdesk@forumias.academy
Page 22 of 38

SFG 2023 | LEVEL 1 | Test #5 – Solutions |


Q.29) With reference to the position of Legislative Council of a State and the Rajya Sabha under the Indian
Constitution, consider the following Statements:
1. Like the Rajya Sabha, the Legislative Councils cannot reject or amend a Money Bill.
2. Unlike the Rajya Sabha, the Legislative Council does not participate in the election of the President of
India.
3. The Legislative Council does not have any exclusive or special powers as enjoyed by the Rajya Sabha.
Which of the Statements given above are correct?
a) 1 and 2 only
b) 2 and 3 only
c) 1 and 3 only
d) 1, 2 and 3

Ans) d
Exp) Option d is the correct answer.
Even though both the council and the Rajya Sabha are second chambers, the Constitution has given the
council much lesser importance than the Rajya Sabha.
Statement 1 is correct: A Money Bill can be introduced only in the assembly and not in the legislative
council. The Legislative councils, like the Rajya Sabha, cannot amend or reject a money bill. It should return
the bill to the assembly within 14 days, either with recommendations or without recommendations.
Similarly, a Money Bill can be introduced only in the Lok Sabha and not in the Rajya Sabha. Rajya Sabha
cannot amend or reject a Money Bill. It should return the bill to the Lok Sabha within 14 days, either with
recommendations or without recommendations
Statement 2 is correct: The Council does not participate in the election of the President of India and
representatives of the state in the Rajya Sabha. The council has no effective say in the ratification of a
constitutional amendment bill. In this respect also, the will of the assembly prevails over that of the council.
Statement 3 is correct: The Rajya Sabha has been given four exclusive or special powers that are not
enjoyed by the Legislative Council of a State: 1. It can authorise the Parliament to make a law on a subject
enumerated in the State List (Article 249). 2. It can authorise the Parliament to create new All-India Services
common to both the Centre and states (Article 312). 3. It alone can initiate a move for the removal of the
vice president. 4. A proclamation can remain effective even if it is approved by the Rajya Sabha alone
(Articles 352, 356 and 360).
Source: 546, 725-728, M. Laxmikant 6th Edition.pdf

Q.30) Consider the following statements with reference to Magnetars:


1. A Magnetar is a type of neutron star that has an ultra-powerful magnetic field.
2. Magnetars can be more luminous than Sun even in an inactive state.
Which of the statements given above is/are correct?
a) 1 only
b) 2 only
c) Both 1 and 2
d) Neither 1 nor 2

Ans) c
Exp) Option c is the correct answer.
Statement 1 is correct. A magnetar is extremely dense type of neutron star, and its defining feature that
it has an ultra-powerful magnetic field, trillions of times stronger than sun.

Forum Learning Centre: Delhi - 2nd Floor, IAPL House, 19 Pusa Road, Karol Bagh, New Delhi - 110005 | Patna - 2nd floor, AG Palace, E Boring Canal Road,
Patna, Bihar 800001 | Hyderabad - 1st & 2nd Floor, SM Plaza, RTC X Rd, Indira Park Road, Jawahar Nagar, Hyderabad, Telangana 500020
9821711605 | https://academy.forumias.com | admissions@forumias.academy | helpdesk@forumias.academy
Page 23 of 38

SFG 2023 | LEVEL 1 | Test #5 – Solutions |


Statement 2 is correct. Even in an inactive state, magnetars can be one hundred thousand times more
luminous than our Sun. The energy that was released is equivalent to that which our Sun radiates in one
hundred thousand years. Magnetars can cause starquakes, or earthquakes on stars.
Source: https://www.sciencealert.com/a-dead-star-has-erupted-with-all-the-fire-and-fury-of-100-
000-
suns#:~:text=%22Even%20in%20an%20inactive%20state,astrophysicist%20Alberto%20J.%20Castro%2D
Tirado
https://www.thehindu.com/sci-tech/What-are-magnetars/article14640121.ece

Q.31) Out of the following statements, choose the one that brings out the principle underlying the Cabinet
form of Government:
a) An arrangement for minimizing the criticism against the Government whose responsibilities are complex
and hard to carry out the satisfaction of all.
b) A mechanism for speeding up the activities of the Government whose responsibilities are increasing day
by day.
c) A mechanism of parliamentary democracy for ensuring collective responsibility of the Government to
the people.
d) A device for strengthening the hands of the head of the Government whose hold over the people is in a
state of decline.

Ans) c
Exp) Option c is the correct answer.
Parliamentary democracy envisages a cabinet form of government with a President as a titular head (and a
governor in the states) with responsibility of ministers to the Parliament.
One of the key features of Parliamentary System of Government is that the Executive is responsible to the
Legislature. Thus, Under Article 75(3), In India the Council of Ministers is collectively responsible to the
House of People (Lok Sabha). The Council of Ministers remains in office as long as they enjoy the support
and confidence of the Lok Sabha.
The features of parliamentary government in India are:
(a) Presence of nominal and real executives;
(b) Majority party rule,
(c) Collective responsibility of the executive to the legislature,
(d) Membership of the ministers in the legislature,
(e) Leadership of the Prime Minister or the Chief Minister,
(f) Dissolution of the lower House (Lok Sabha or Assembly).
Source) UPSC CSE 2017

Q.32) With reference to the Secretariat of Parliament in India, consider the following statements:
1. One of its functions is to ensure payment of salary and other allowances to the Members of the House.
2. It is headed by Minister of Parliamentary Affairs.
3. The recruitment and service conditions of the secretarial staff are regulated by the Council of Ministers.
Which of the statements given above is/are correct?
a) 1 only
b) 2 only
c) 1 and 3 only
d) 2 and 3 only

Forum Learning Centre: Delhi - 2nd Floor, IAPL House, 19 Pusa Road, Karol Bagh, New Delhi - 110005 | Patna - 2nd floor, AG Palace, E Boring Canal Road,
Patna, Bihar 800001 | Hyderabad - 1st & 2nd Floor, SM Plaza, RTC X Rd, Indira Park Road, Jawahar Nagar, Hyderabad, Telangana 500020
9821711605 | https://academy.forumias.com | admissions@forumias.academy | helpdesk@forumias.academy
Page 24 of 38

SFG 2023 | LEVEL 1 | Test #5 – Solutions |


Ans) a
Exp) Option a is the correct answer.
Each House of Parliament has separate secretarial staff of its own under Article 98, though there can be
some posts common to both the Houses. The Secretariat is an independent body which functions under
the ultimate guidance and control of the Presiding Officer.
Statement 1 is correct: The main activities of the Secretariat include the following: -
1) providing secretarial assistance and support to the effective functioning of the House.
2) payment of salary and other allowances to the Members of the House.
3) providing amenities as admissible to Members of the House.
4) servicing the various Parliamentary Committees.
5) preparing research and reference material and bringing out various publications.
6) recruitment of manpower in the Rajya Sabha Secretariat and attending to personnel matters. preparing
and publishing a record of the day-to-day proceedings of the House and
7) bringing out such other publications, as may be required concerning the functioning of the House.
Statement 2 is incorrect: The secretariat of each House is headed by a secretary-general (not the Minister
of Parliamentary Affairs). He is a permanent officer and is appointed by the presiding officer of the House.
Statement 3 is incorrect: Parliament may by law regulate the recruitment and the conditions of service of
persons appointed to the secretarial staff of either House of Parliament.
Knowledge Base:
In the discharge of his Constitutional and statutory responsibilities, the Presiding Officer is assisted by the
Secretary-General (whose pay scale, position and status etc. is equivalent to that of the highest ranking
official in the Government of India i.e., Cabinet Secretary), functionaries of the level of the Additional
Secretary, Joint Secretary and other officers and staff of the Secretariat at various levels. These are not
regulated by the Council of Ministers.
Source: Indian Polity by Laxmikant – 6th Edition – Chapter 22 Parliament.
https://rajyasabha.nic.in/RajyaSabhaSecretariat

Q.33) With reference to the Parliament in India, consider the following statements:
1. A bill passed by the Rajya Sabha but pending in the Lok Sabha does not lapse on the dissolution of the Lok
Sabha
2. A bill passed by the Lok Sabha but pending in the Rajya Sabha does not lapse on the dissolution of the
Lok Sabha.
3. Prorogation of the House does not affect the bills pending before the House.
Which of the statements given above is/are correct?
a) 1 and 3 only
b) 3 only
c) 1 and 2 only
d) 2 and 3 only

Ans) b
Exp) Option b is the correct answer.
Dissolution ends the very life of the existing House, and a new House is constituted after general elections
are held.
Statements 1 and 2 are incorrect: When the Lok Sabha is dissolved, all business including bills, motions,
resolutions, notices, petitions and so on pending before it or its committees lapse. However, some pending
bills and all pending assurances that are to be examined by the Committee on Government Assurances do
not lapse on the dissolution of the Lok Sabha. The position with respect to lapsing of bills is as follows:

Forum Learning Centre: Delhi - 2nd Floor, IAPL House, 19 Pusa Road, Karol Bagh, New Delhi - 110005 | Patna - 2nd floor, AG Palace, E Boring Canal Road,
Patna, Bihar 800001 | Hyderabad - 1st & 2nd Floor, SM Plaza, RTC X Rd, Indira Park Road, Jawahar Nagar, Hyderabad, Telangana 500020
9821711605 | https://academy.forumias.com | admissions@forumias.academy | helpdesk@forumias.academy
Page 25 of 38

SFG 2023 | LEVEL 1 | Test #5 – Solutions |


1) A bill pending in the Lok Sabha lapses (whether originating in the Lok Sabha or transmitted to it by
the Rajya Sabha).
2) A bill passed by the Lok Sabha but pending in the Rajya Sabha lapses.
3) A bill not passed by the two Houses due to disagreement and if the president has notified the holding of
a joint sitting before the dissolution of Lok Sabha, does not lapse.
4) A bill pending in the Rajya Sabha but not passed by the Lok Sabha does not lapse.
5) A bill passed by both Houses but pending assent of the president does not lapse.
6) A bill passed by both Houses but returned by the president for reconsideration of Houses does not lapse.
Statement 3 is correct: Prorogation terminates both the sitting and the session of the House. It is done by
the President of India. Prorogation does not affect the bills or any other business pending before the
House. However, all pending notices (other than those for introducing bills) lapse on prorogation and fresh
notices have to be given for the next session.
Source: Indian Polity by Laxmikant – 6th Edition – Chapter 22 Parliament.

Q.34) With reference to Indian Parliament, Consider the following statements about Department Related
Standing Committees (DRSCs):
1. They are constitutional bodies mentioned in Part V of the Constitution of India.
2. The term of office of each standing committee is coterminous with the life of Lok Sabha.
3. They secure accountability of the Council of Ministers towards the Parliament.
4. They assist the Parliament in scrutinizing the bills referred to it.
Which of the statements given above is/are correct?
a) 2, 3 and 4 only
b) 3 only
c) 3 and 4 only
d) 1, 2, 3 and 4

Ans) c
Exp) Option c is the correct answer.
Each Department Relates Standing Committee consists of 31 members (21 from Lok Sabha and 10 from Rajya
Sabha). The members of the Lok Sabha are nominated by the Speaker, just as the members of the Rajya
Sabha are nominated by the Chairman from amongst its members. A minister is not eligible to be nominated
as a member of any of the standing committees. In case a member, after his nomination to any of the
standing committees, is appointed a minister, he then ceases to be a member of the committee
Statement 1 is incorrect: They are not constitutional bodies. On the recommendation of the Rules
Committee of the Lok Sabha, 17 DRSCs were set up in the Parliament in 1993. In 2004, seven more such
committees were set up, thus increasing their number from 17 to 24. Out of the 24 Departmental Standing
Committees, 8 work under the Rajya Sabha and 16 under the Lok Sabha.
Statement 2 is incorrect: The term of office of each standing committee is one year from the date of its
constitution.
Statement 3 is correct: They secure more accountability of the Council of Ministers to the Parliament.
Through Committees, Parliament exercises its control and influence over administration and keeps
vigilance over the executive.
Statement 4 is correct: The Committees aid and assist the Legislature in discharging its duties and
regulating its functions effectively, expeditiously and efficiently. They assist the Parliament in thoroughly
and systematically scrutinizing the bills and matters which could not be discussed on the floor at length.
These bills are referred to the Committee by the Chairman or the Speaker. The Committees also provide
the expertise on a matter which is referred to them.

Forum Learning Centre: Delhi - 2nd Floor, IAPL House, 19 Pusa Road, Karol Bagh, New Delhi - 110005 | Patna - 2nd floor, AG Palace, E Boring Canal Road,
Patna, Bihar 800001 | Hyderabad - 1st & 2nd Floor, SM Plaza, RTC X Rd, Indira Park Road, Jawahar Nagar, Hyderabad, Telangana 500020
9821711605 | https://academy.forumias.com | admissions@forumias.academy | helpdesk@forumias.academy
Page 26 of 38

SFG 2023 | LEVEL 1 | Test #5 – Solutions |


Knowledge Base:
These DRSCs are entrusted with the following functions: -
(a) to consider the Demands for Grants of the related Ministries/Departments and report thereon. The
report shall not suggest anything of the nature of cut motions;
(b) to examine Bills, pertaining to the related Ministries/Departments, referred to the Committee by the
Chairman or the Speaker, as the case may be, and report thereon;
(c) to consider the annual reports of the Ministries/Departments and report thereon; and
(d) to consider national basic long term policy documents presented to the Houses, if referred to the
Committee by the Chairman or the Speaker, as the case may be, and report thereon.
Source: https://rajyasabha.nic.in/Committees/DepartmentRelatedSC_RS?id=17

Q.35) Which among the following statements is/are correct with regards to Type 1 Diabetes?
1. It accounts for the vast majority (over 90%) of diabetes worldwide.
2. It is the major type of diabetes in childhood but can occur at any age.
Select the correct answer using the code given below:
a) 1 only
b) 2 only
c) Both 1 and 2
d) Neither 1 nor 2

Ans) b
Exp) Option b is the correct answer.
Diabetes is a serious chronic condition that occurs when the body cannot produce enough insulin or cannot
effectively use the insulin it does produce.
The 10th edition of the International Diabetes Federation Diabetes Atlas has been released. It has been
reported that Diabetes is one of the fastest-growing global health emergencies of the 21st century.
Type 1 diabetes is a condition in which your immune system destroys insulin-making cells in the pancreas.
The condition is usually diagnosed in children and young people, so it used to be called juvenile diabetes
Statement 1 is incorrect: Type 2 Diabetes accounts for the vast majority (and not Type 1) of diabetes
worldwide. Type 2 diabetes is a lifelong disease that keeps your body from using insulin the way it should.
People with type 2 diabetes are said to have insulin resistance
Statement 2 is correct: Type 1 diabetes is the major type of diabetes in childhood but can occur at any
age. It cannot be cured, people with type 1 diabetes require insulin to survive.
Knowledge Base:
1) In 2021, one in 10 adults worldwide, which means around 537 million people are now living with diabetes.
This is higher than the 463 million adults who lived with the condition in 2019.
2) Diabetes now ranks among the top 10 causes of global mortality, responsible for an estimated 6.7 million
deaths in 2021. Furthermore, Globally, 88% of adults living with undiagnosed diabetes are in low- and
middle-income countries.
Source: https://www.downtoearth.org.in/blog/health/a-new-report-shows-worrying-growth-of-the-
diabetes-pandemic-80558

Forum Learning Centre: Delhi - 2nd Floor, IAPL House, 19 Pusa Road, Karol Bagh, New Delhi - 110005 | Patna - 2nd floor, AG Palace, E Boring Canal Road,
Patna, Bihar 800001 | Hyderabad - 1st & 2nd Floor, SM Plaza, RTC X Rd, Indira Park Road, Jawahar Nagar, Hyderabad, Telangana 500020
9821711605 | https://academy.forumias.com | admissions@forumias.academy | helpdesk@forumias.academy
Page 27 of 38

SFG 2023 | LEVEL 1 | Test #5 – Solutions |


Q.36) Which of the following pairs are correctly matched?
List I List II
Cut motions Description
1. Policy Cut The amount of the demand
Motion be reduced by ₹100
2. Token Cut The amount of the demand
Motion be reduced to Rs 1
3. Economy The amount of the demand
Cut Motion be reduced by a specified
amount
Select the correct answer using the code given below:
a) 1 only
b) 2 and 3 only
c) 3 only
d) 1 and 2 only

Ans) c
Exp) Option c is the correct answer.
Each demand of budget is voted separately by the Lok Sabha. The members of Parliament can discuss the
details of the budget during this stage. They can also move motions to reduce any demand for grant. Such
motions are called as ‘cut motion’.
Pair 1 is incorrect: Policy cut motion represents the disapproval of the policy underlying the demand. It
states that the amount of the demand be reduced to Re 1(and not reduced by ₹100). The members can
also advocate an alternative policy. Policy cut motion represents the disapproval of the policy undermining
the demand. However, if a member moves the cut, they have to indicate in precise terms the details of the
policy which they want to discuss and should be confined to the specific points mentioned in the cut notice.
Pair 2 is incorrect: Token Cut Motion ventilates a specific grievance that is within the sphere of
responsibility of the Government of India. It states that the amount of the demand be reduced by ₹100
(and not to be reduced to Re 1).
Pair 3 is correct: Economy Cut Motion represents the economy that can be affected in the proposed
expenditure. It states that the amount of the demand be reduced by a specified amount (which may be
either a lump sum reduction in the demand or omission or reduction of an item in the demand).
Source: https://www.business-standard.com/about/what-is-cut-motion
Indian polity by M Laxmikanth pdf 6th edition. Chapter name-Parliament. Page no-541.

Q.37) Consider the following statements with reference to the Joint Sitting of the Parliament:
1. It is governed by the Rules of Procedure of both Lok Sabha and Rajya Sabha.
2. No new amendments can be proposed to a bill at a joint sitting.
3. The bill is deemed to have been passed only if it is passed by a majority of the total membership of both
the houses of parliament.
Which of the statements given above is/are correct?
a) 1 and 2 only
b) 2 only
c) 1 and 3 only
d) None of the above

Ans) d
Exp) Option d is the correct answer.

Forum Learning Centre: Delhi - 2nd Floor, IAPL House, 19 Pusa Road, Karol Bagh, New Delhi - 110005 | Patna - 2nd floor, AG Palace, E Boring Canal Road,
Patna, Bihar 800001 | Hyderabad - 1st & 2nd Floor, SM Plaza, RTC X Rd, Indira Park Road, Jawahar Nagar, Hyderabad, Telangana 500020
9821711605 | https://academy.forumias.com | admissions@forumias.academy | helpdesk@forumias.academy
Page 28 of 38

SFG 2023 | LEVEL 1 | Test #5 – Solutions |


As per Article 108 of Constitution a Joint sitting is an extraordinary mechanism provided by the
Constitution of India to resolve a deadlock between the two Houses over the passage of a bill. It is to be
noted that the provisions of Joint seating are applicable to ordinary bills or financial bills only. Not all
bills can be referred to a joint sitting of Parliament. There are two exceptions.
1) Money bill under Article 110.
2) Constitution amendment bill under Article 368.
Statement 1 is incorrect: The joint sitting is governed by the Rules of Procedure of Lok Sabha and not of
Rajya Sabha. The quorum to constitute a joint sitting is one-tenth of the total number of members of the
two Houses.
Statement 2 is incorrect:
The Constitution has specified that at a joint sitting, new amendments to the bill cannot be proposed except
in two cases:
1) those amendments that have caused final disagreement between the Houses; and
2) those amendments that might have become necessary due to the delay in the passage of the bill.
Statement 3 is incorrect: The bill in dispute in joint sitting of the two houses is deemed to have been
passed by a majority of the total number of members of both the Houses present and voting (and not by
a majority of the total membership of both the houses of parliament). Normally, the Lok Sabha with
greater number wins the battle in a joint sitting.
Source: Indian polity by M Laxmikanth pdf 6th edition. Chapter name-Parliament. Page no-534 and 535.

Q.38) Consider the following statements with reference to the Constitutional provisions regarding the
Budget:
1. Parliament does not have right either to discuss or to vote on the expenditures charged on the
Consolidated Fund of India.
2. The term ‘budget’ has nowhere been used in the Constitution of India.
3. With regard to the enactment of budget, Parliament can reduce or abolish a tax but cannot increase it.
4. Vote on the demands for grants is an exclusive privilege of the Lok Sabha and Rajya Sabha has no power
regarding it.
Which of the statements given above are correct?
a) 1, 2 and 3 only
b) 2, 3 and 4 only
c) 1, 3 and 4 only
d) 1, 2, 3 and 4

Ans) b
Exp) Option b is the correct answer.
Article 112 of the Constitution deals with the budget which is a statement of the estimated receipts and
expenditure of the Government of India in a financial year.
Statement 1 is incorrect: According to the constitutional provisions, the budget expenditure charged on
the Consolidated Fund of India shall not be submitted for the vote of Parliament. However, it can be
discussed by the Parliament.
Statement 2 is correct: The Constitution refers to the budget as the ‘annual financial statement’. The
term ‘budget’ has nowhere been used in the Constitution of India.
Statement 3 is correct: The Constitution of India contains the following provisions with regard to the
enactment of budget:
1) Parliament can reduce or abolish a tax but cannot increase it.

Forum Learning Centre: Delhi - 2nd Floor, IAPL House, 19 Pusa Road, Karol Bagh, New Delhi - 110005 | Patna - 2nd floor, AG Palace, E Boring Canal Road,
Patna, Bihar 800001 | Hyderabad - 1st & 2nd Floor, SM Plaza, RTC X Rd, Indira Park Road, Jawahar Nagar, Hyderabad, Telangana 500020
9821711605 | https://academy.forumias.com | admissions@forumias.academy | helpdesk@forumias.academy
Page 29 of 38

SFG 2023 | LEVEL 1 | Test #5 – Solutions |


2) The President shall in respect of every financial year cause to be laid before both the Houses of
Parliament a statement of estimated receipts and expenditure of the Government of India for that year.
3) No demand for a grant shall be made except on the recommendation of the President.
4) No money shall be withdrawn from the Consolidated Fund of India except under appropriation made by
law.
5) No money bill imposing tax shall be introduced in the Parliament except on the recommendation of the
President, and such a bill shall not be introduced in the Rajya Sabha.
Statement 4 is correct: The Constitution of India has defined the relative roles or position of both the
Houses of Parliament with regard to the enactment of the budget in the following way:
1) A money bill or finance bill dealing with taxation cannot be introduced in the Rajya Sabha–it must be
introduced only in the Lok Sabha.
2) The Rajya Sabha has no power to vote on the demand for grants; it is the exclusive privilege of the
Lok Sabha.
3) The Rajya Sabha should return the Money bill (or Finance bill) to the Lok Sabha within fourteen days.
The Lok Sabha can either accept or reject the recommendations made by Rajya Sabha in this regard.
Source: Indian polity by M Laxmikanth pdf 6th edition. Chapter name-Parliament. Page no-536 and 537.

Q.39) With reference to Private Member’s Bills and Public Bills, Consider the following statements:
1. Both types of bills can be introduced in the Parliament after only seven days' notice.
2. No private member’s bill has been passed by Parliament of India till date.
3. A bill introduced by an elected member of Parliament is called as public bill, while a bill introduced by a
nominated member is called as private bill.
Which of the statements given above is/are incorrect?
a) 1 and 2 only
b) 2 and 3 only
c) 1 only
d) 1, 2 and 3

Ans) d
Exp) Option d is the correct answer.
Statement 1 is incorrect: The introduction of Public Bills in the House requires seven days’ notice. The
introduction of Private Bills in the House requires one month’s notice.
Statement 2 is incorrect: The last time a private member’s bill was passed by both Houses was in 1970.It
was the Supreme Court (Enlargement of Criminal Appellate Jurisdiction) Bill, 1968. 14 private member’s
bills — five of which were introduced in Rajya Sabha — have become law so far. Some other private member
bills that have become laws include:
1) Proceedings of Legislature (Protection of Publication) Bill, 1956, in the Lok Sabha;
2) The Salaries and Allowances of Members of Parliament (Amendment) Bill, 1964, introduced by in the Lok
Sabha and
3) The Indian Penal Code (Amendment) Bill, 1967 introduced in the Rajya Sabha. Etc.
Statement 3 is incorrect: Private member's bill is introduced by any member of Parliament other than a
minister (not necessarily a nominated member) while the public bill is introduced by a minister (and not
any other elected member). Public bill reflects of the policies of the government. Private member's bill
reflects the stand of opposition party or a member of parliament on public matter.
Source: Indian polity by M Laxmikanth pdf 6th edition. Chapter name-Parliament. Page no-526.

Forum Learning Centre: Delhi - 2nd Floor, IAPL House, 19 Pusa Road, Karol Bagh, New Delhi - 110005 | Patna - 2nd floor, AG Palace, E Boring Canal Road,
Patna, Bihar 800001 | Hyderabad - 1st & 2nd Floor, SM Plaza, RTC X Rd, Indira Park Road, Jawahar Nagar, Hyderabad, Telangana 500020
9821711605 | https://academy.forumias.com | admissions@forumias.academy | helpdesk@forumias.academy
Page 30 of 38

SFG 2023 | LEVEL 1 | Test #5 – Solutions |


Q.40) Consider the following pairs in context of places in news:
Place in news Country/Region
1. Barbados Caribbean Islands
2. Donbas Poland
3. Amami Islands China
Which of the pairs given above is/are correctly matched?
a) 1 only
b) 1 and 2 only
c) 1 and 3 only
d) 2 and 3 only

Ans) a
Exp) Option a is the correct answer.
Pair 1 is correctly matched: Barbados is an island in the Caribbean region of the Americas, and the most
easterly of the Caribbean Islands. It recently became the newest republic in the world. This is the first time
since the 1970s that a Caribbean state has become a republic.

Pair 2 is incorrectly matched: Donbas is a small region in eastern Ukraine (and not Poland) adjoining the
Russian border. With Russia mobilizing thousands of troops on its border with Ukraine, which has sparked
fears of war, Donbas is once again at the center of a looming conflict.

Pair 3 is incorrectly matched: The Amami Islands is an archipelago near East China Sea that belongs to
Kagoshima Prefecture, Japan (and not China). Recently, sighting of an unidentified submarine believed to
be Chinese was reported by Japan.

Forum Learning Centre: Delhi - 2nd Floor, IAPL House, 19 Pusa Road, Karol Bagh, New Delhi - 110005 | Patna - 2nd floor, AG Palace, E Boring Canal Road,
Patna, Bihar 800001 | Hyderabad - 1st & 2nd Floor, SM Plaza, RTC X Rd, Indira Park Road, Jawahar Nagar, Hyderabad, Telangana 500020
9821711605 | https://academy.forumias.com | admissions@forumias.academy | helpdesk@forumias.academy
Page 31 of 38

SFG 2023 | LEVEL 1 | Test #5 – Solutions |

Source: https://www.thehindu.com/news/international/barbados-says-goodbye-to-queen-elizabeth-
ii-transforms-into-republic/article37765047.ece
https://www.thehindu.com/news/international/donbas-at-the-centre-of-the-ukraine-
crisis/article37935114.ece
https://www.thehindu.com/news/international/japan-says-suspected-chinese-submarine-seen-near-
territorial-waters/article36411503.ece
https://the-japan-news.com/news/article/0008134870

Q.41) Consider the following statements:


1. In the election for Lok Sabha or State Assembly, the winning candidate must get at least 50 percent of
the votes polled, to be declared elected.
2. According to the provisions laid down in the Constitution of India, in Lok Sabha, the Speaker’s post goes
to the majority party and the Deputy Speaker’s to the Opposition.
Which of the statements given above is/are correct?
a) 1 only
b) 2 only
c) Both 1 and 2
d) Neither 1 nor 2

Ans) d
Exp) Option d is the correct answer.
Statement 1 is incorrect. As India follows the First Past the Post System, where a candidate who polls more
votes than any other candidate is declared elected.
Statement 2 is incorrect. As per the constitution, the speaker and deputy speaker in Lok Sabha are elected
among its members. It is not a constitutional provision but a convention that Speaker’s post goes to the
majority party and the Deputy Speaker’s to the Opposition.
Source) UPSC CSE 2017

Q.42) Consider the following statements with reference to the finance bill and appropriation bill:
1. A Finance Bill is a Money Bill.
2. The government of India cannot withdraw money from the Consolidated Fund of India till the enactment
of the appropriation bill.

Forum Learning Centre: Delhi - 2nd Floor, IAPL House, 19 Pusa Road, Karol Bagh, New Delhi - 110005 | Patna - 2nd floor, AG Palace, E Boring Canal Road,
Patna, Bihar 800001 | Hyderabad - 1st & 2nd Floor, SM Plaza, RTC X Rd, Indira Park Road, Jawahar Nagar, Hyderabad, Telangana 500020
9821711605 | https://academy.forumias.com | admissions@forumias.academy | helpdesk@forumias.academy
Page 32 of 38

SFG 2023 | LEVEL 1 | Test #5 – Solutions |


3. Unlike the Appropriation Bill, the amendments seeking to reject or reduce a tax can be moved in the case
of finance bill.
Which of the statements given above is/are correct?
a) 1 and 2 only
b) 2 only
c) 1 and 3 only
d) 1, 2 and 3

Ans) d
Exp) Option d is the correct answer.
Statement 1 is correct: A Finance Bill is a Money Bill as defined in Article 110 of the Constitution. It is
accompanied by a Memorandum explaining the provisions included in it. The Finance Bill is introduced
to give effect to the financial proposals of the Government of India for the following year. At the time of
presentation of the Annual Financial Statement before Parliament, a Finance Bill is also presented in
fulfillment of the requirement of Article 110 (1)(a) of the Constitution, detailing the imposition, abolition,
remission, alteration or regulation of taxes proposed in the Budget.
Statement 2 is correct: Appropriation act authorizes the payments from the Consolidated Fund of India.
This means that the government cannot withdraw money from the Consolidated Fund of India till the
enactment of the appropriation bill. Under Article 114(3) of the Constitution, no amount can be withdrawn
from the Consolidated Fund without the enactment of appropriation bill by Parliament. The Appropriation
Bill becomes the Appropriation Act after President gives its assent to the bill.
Statement 3 is correct: The Finance Bill is introduced to give effect to the financial proposals of the
Government of India for the following year. It is subjected to all the conditions applicable to a Money Bill.
Unlike the Appropriation Bill, the amendments seeking to reject or reduce a tax can be moved in the
case of finance bill. According to the Provisional Collection of Taxes Act of 1931, the Finance Bill must be
enacted within 75 days.
Source: Indian polity by M Laxmikanth pdf 6th edition. Chapter name-Parliament. Page no-543.
https://www.indiabudget.gov.in/budget2011-2012/ub2011-12/keybud/keybud2011.pdf

Q.43) With reference to the parliamentary committees in parliamentary system of government, Consider
the following statements:
1. The Constitution of India makes no mention of parliamentary committees.
2. Parliamentary committees provide a forum for building consensus across political parties.
3. Consultative Committees are a type of Parliamentary committees.
Which of the statements given above is/are correct?
a) 1 and 2 only
b) 2 only
c) 2 and 3 only
d) 1, 2 and 3

Ans) b
Exp) Option b is the correct answer.
Statement 1 is incorrect: The Constitution of India makes a mention of Parliamentary Committees at
different places, but without making any specific provisions regarding their composition, tenure,
functions, etc. All these matters are dealt by the rules of two Houses.
Statement 2 is correct: Parliamentary Committees provide a forum for building consensus across
political parties. The proceedings of the House during sessions are televised, and MPs are likely to stick to

Forum Learning Centre: Delhi - 2nd Floor, IAPL House, 19 Pusa Road, Karol Bagh, New Delhi - 110005 | Patna - 2nd floor, AG Palace, E Boring Canal Road,
Patna, Bihar 800001 | Hyderabad - 1st & 2nd Floor, SM Plaza, RTC X Rd, Indira Park Road, Jawahar Nagar, Hyderabad, Telangana 500020
9821711605 | https://academy.forumias.com | admissions@forumias.academy | helpdesk@forumias.academy
Page 33 of 38

SFG 2023 | LEVEL 1 | Test #5 – Solutions |


their party positions on most matters. Committees have closed door meetings, which allow them to freely
question and discuss issues and arrive at a consensus.
Statement 3 is incorrect A parliamentary committee means a committee that:
1) Is appointed or elected by the House or nominated by the Speaker / Chairman1
2) Works under the direction of the Speaker / Chairman
3) Presents its report to the House or to the Speaker /Chairman
4) Has a secretariat provided by the Lok Sabha / Rajya Sabha
The consultative committees, which also consist of members of Parliament, are not parliamentary
committees as they do not fulfil above four conditions.
Source: https://prsindia.org/theprsblog/importance-parliamentary-committees
Indian polity by M Laxmikanth pdf 6th edition. Chapter name-Parliamentary committees. Page no-576.

Q.44) The origin of this committee can be traced back to the standing financial committee set up in 1921. It
was first constituted in the post-independence era on the recommendation of John Mathai. Originally the
committee had 25 members but in 1956 its membership was raised to 30. All the members of this committee
are from Lok Sabha only. The Rajya Sabha has no representation in this committee.
a) Public Accounts Committee
b) Estimates Committee
c) Committee on Public Undertakings
d) Business Advisory Committee

Ans) b
Exp) Option b is the correct answer.
Option a is incorrect: Public Accounts Committee was set up first in 1921 under the provisions of the
Government of India Act of 1919. At present, it consists of 22 members (15 from the Lok Sabha and 7 from
the Rajya Sabha). The members are elected by the Parliament every year from amongst its members
according to the principle of proportional representation by means of the single transferable vote. Thus,
all parties get due representation in it. The term of office of the members is one year. A minister cannot be
elected as a member of the committee.
Option b is correct: The origin of Estimates Committee can be traced to the standing financial committee
set up in 1921. The first Estimates Committee in the post-independence era was constituted in 1950 on
the recommendation of the then finance minister John Mathai. Originally the committee had 25 members
but in 1956 its membership was raised to 30. All the thirty members of estimates committee are from Lok
Sabha only. The Rajya Sabha has no representation in this committee. These members are elected by the
Lok Sabha every year from amongst its own members, according to the principles of proportional
representation by means of a single transferable vote. Thus, all parties get due representation in it. The
function of the committee is to examine the estimates included in the budget and suggest ‘economies’ in
public expenditure. Hence, it has been described as a ‘continuous economy committee’.
Option c is incorrect: Committee on Public Undertakings was created in 1964 on the recommendation
of the Krishna Menon Committee. Originally the committee had 15 members (10 from the Lok Sabha and
5 from the Rajya Sabha). However, in 1974, its membership was raised to 22 (15 from the Lok Sabha and 7
from the Rajya Sabha). The members of this committee are elected by the Parliament every year from
amongst its own members according to the principle of proportional representation by means of a single
transferable vote. Thus, all parties get due representation in it. The term of office of the members is one
year. A minister cannot be elected as a member of the committee.
Option d is incorrect: Business Advisory Committee regulates the programme and time table of the
House. It allocates time for the transaction of legislative and other business brought before the House by

Forum Learning Centre: Delhi - 2nd Floor, IAPL House, 19 Pusa Road, Karol Bagh, New Delhi - 110005 | Patna - 2nd floor, AG Palace, E Boring Canal Road,
Patna, Bihar 800001 | Hyderabad - 1st & 2nd Floor, SM Plaza, RTC X Rd, Indira Park Road, Jawahar Nagar, Hyderabad, Telangana 500020
9821711605 | https://academy.forumias.com | admissions@forumias.academy | helpdesk@forumias.academy
Page 34 of 38

SFG 2023 | LEVEL 1 | Test #5 – Solutions |


the government. The Lok Sabha committee consists of 15 members including the Speaker as its chairman.
In the Rajya Sabha, it has 11 members including the Chairman as its ex officio chairman.
Source: Indian polity by M Laxmikanth pdf 6th edition. Chapter name-Parliamentary committees. Page no-
580,582,583,594.

Q.45) Which among the following statements is/are incorrect with regards to Media Regulation in India:
1. Press Council of India is assigned with the authority to regulate all kind of media coverage including Print,
Radio and TV news.
2. The News Broadcasters and Digital Association (NBDA) is a self-regulatory body for the digital
broadcasters.
Select the correct answer using the code given below:
a) 1 only
b) 2 only
c) Both 1 and 2
d) Neither 1 nor 2

Ans) a
Exp) Option a is the correct answer.
Ministry of Broadcasting and Information oversees both print and digital media in India. There are also
some autonomous bodies that keep eyes on specific sections of the media.
Statement 1 is incorrect: Press Council of India is vested with statutory powers for preserving the
freedom of Press and for maintaining and improving the standards of newspapers, print media, and radio
(and not TV news). It is established under Press Council Act, 1978.
Statement 2 is correct: News Broadcasters & Digital Standards Association administers broadcastings,
which has been voluntarily drawn by the NBDA (News Broadcasters & Digital Association) for its member
broadcasters to demonstrate their commitment to responsible broadcasting and to self-regulate
themselves. It is the collective voice of the news, current affairs and digital broadcasters in India. It is an
organization funded entirely by its members.
Knowledge Base:
1) Recently, the Parliamentary Standing Committee on Communications and Information Technology
has submitted two reports titled “Ethical Standards in Media Coverage” and “Suspension `of Telecom
Services/Internet and Its Impact” to the Parliament.
2) One of the key recommendations is that a ‘Media Council’ should be set up with statutory powers to
enforce its powers to check “irregularities” in print, electronic and digital media platforms. This is
because existing regulatory bodies like the Press Council of India (PCI) and News Broadcasting and
Digital Standards Authority (NBDSA) have limited efficacy because they do not have the powers to
enforce their decisions.
Source:
http://164.100.47.193/lsscommittee/Communications%20and%20Information%20Technology/17_Com
munications_and_Information_Technology_27.pdf
http://www.nbanewdelhi.com/about-nba

Q.46) Consider the following statements with reference to the Consultative Committees:
1. These committees are headed by a member of parliament other than a minister.
2. These committees provide a forum for informal discussions between the ministers and the members of
Parliament on policies of the government.
3. Consultative committees are constituted by the Ministry of Home Affairs.

Forum Learning Centre: Delhi - 2nd Floor, IAPL House, 19 Pusa Road, Karol Bagh, New Delhi - 110005 | Patna - 2nd floor, AG Palace, E Boring Canal Road,
Patna, Bihar 800001 | Hyderabad - 1st & 2nd Floor, SM Plaza, RTC X Rd, Indira Park Road, Jawahar Nagar, Hyderabad, Telangana 500020
9821711605 | https://academy.forumias.com | admissions@forumias.academy | helpdesk@forumias.academy
Page 35 of 38

SFG 2023 | LEVEL 1 | Test #5 – Solutions |


Which of the statements given above is/are correct?
a) 2 only
b) 2 and 3 only
c) 1 and 3 only
d) 1 and 2

Ans) a
Exp) Option a is the correct answer.
Consultative Committees for various Ministries or Departments are normally constituted after general
election to the Lok Sabha and as and when necessary. The constitution of these committees is generally
timed with the summoning of the Budget Session of Parliament.
Statement 1 is incorrect: Consultative committees of parliament are attached to various ministries or
departments of the Central Government. They consist of members of both the Houses of Parliament. The
Minister / Minister of State in charge of the Ministry concerned acts as the chairman of the consultative
committee of that ministry.
Statement 2 is correct: Consultative Committees provide a forum for informal discussions between the
ministers and the members of Parliament on policies and programmes of the government and the
manner of their implementation. Discussions at the meetings of these committees are held in a free and
frank manner, cutting across party lines.
Statement 3 is incorrect: Consultative Committees are constituted by the Ministry of Parliamentary
Affairs (and not by the ministry of home affairs). The guidelines regarding the composition, functions and
procedures of these committees are formulated by Ministry of Parliamentary Affairs. The Ministry also
makes arrangements for holding their meetings both during the session and the inter-session period of
Parliament.
Source: https://mpa.gov.in/sites/default/files/parlia7.pdf
Indian polity by M Laxmikanth pdf 6th edition. Chapter name-Parliamentary committees. Page no-596.

Q.47) What are the basic functions of Parliament in India?


1. Exercising control over the Council of Ministers
2. Implementation of the Laws
3. Final interpretation of Constitution
4. Amending various provisions of the Constitution
5. Enactment of Annual Financial Statement of Government
Select the correct answer using the code given below:
a) 1, 2 and 5 only
b) 3, 4 and 5 only
c) 1, 4 and 5 only
d) 1, 2, 3, 4 and 5

Ans) c
Exp) Option c is the correct answer.
The Parliament occupies a central position and has a multifunctional role. It has legislative, executive,
financial, constitutional, judicial, electoral powers and functions associated with it.
Statement 1 is correct: The Constitution of India established a parliamentary form of government in which
the Executive is responsible to the Parliament for its policies and acts. Hence, the Parliament exercises
control over the Executive through question-hour, zero hour, half-an-hour discussion, short duration
discussion, calling attention motion, adjournment motion, no-confidence motion, censure motion and

Forum Learning Centre: Delhi - 2nd Floor, IAPL House, 19 Pusa Road, Karol Bagh, New Delhi - 110005 | Patna - 2nd floor, AG Palace, E Boring Canal Road,
Patna, Bihar 800001 | Hyderabad - 1st & 2nd Floor, SM Plaza, RTC X Rd, Indira Park Road, Jawahar Nagar, Hyderabad, Telangana 500020
9821711605 | https://academy.forumias.com | admissions@forumias.academy | helpdesk@forumias.academy
Page 36 of 38

SFG 2023 | LEVEL 1 | Test #5 – Solutions |


other discussions. It also supervises the activities of the Executive with the help of its committees like
committee on government assurance, committee on subordinate legislation, committee on petitions, etc.
Statement 2 is incorrect: The primary function of Parliament is to make laws for the governance of the
country or frame the laws according to current scenario of the society but the implementation of laws is
totally the role of the executive. Executive has responsibility for the governance of a state.
Statement 3 is incorrect: Neither the executive nor the Parliament has the power to be the final interpreter
and protector of the constitution, the power lies in the hand of the Supreme Court. The final interpreter,
protector & guardian of the Indian Constitution is the Supreme Court of India.
Statement 4 is correct: Parliament is vested with the powers to amend the Constitution by way of addition,
variation, or repeal of any provision. The major part of the Constitution can be amended by the Parliament
with special majority, that is, a majority of the total membership of each House and a majority of not less
than two thirds of the members present and voting in each House. Some other provisions of the
Constitution can be amended by the Parliament with simple majority, that is, a majority of the members
present and voting in each House of Parliament. Only a few provisions of the Constitution can be amended
by the Parliament (by special majority) and with the consent of at least half of the state Legislatures (by
simple majority).
Statement 5 is correct: According to Indian Constitution, no tax can be levied or collected and no
expenditure can be incurred by the Executive except under the authority and with the approval of
Parliament. Hence, the budget (Annual Financial Statement) is placed before the Parliament for its
approval. The enactment of the budget by the Parliament legalises the receipts and expenditure of the
government for the ensuing financial year.
Source: M laxmikant (chap 22- Parliament )
NCERT Class VIII: Social and Political Life III Ch 10

Q.48) Consider the following statements with reference to the Contingency Fund of India:
1. Any expenditure incurred from this fund does not require any kind of authorization from the Parliament.
2. The Corpus of this fund has been fixed by the Constitution.
3. The Cabinet Secretary holds the fund on behalf of the President of India.
Which of the statements given above is/are correct?
a) 1 and 2 only
b) 2 only
c) 1 and 3 only
d) None

Ans) d
Exp) Option d is the correct answer.
The Constitution of India provides for the following three kinds of funds for the Central government:
1) Consolidated Fund of India (Article 266)
2) Public Account of India (Article 266)
3) Contingency Fund of India (Article 267)
Statement 1 is incorrect: Article 267 of the Constitution mandates formation of a corpus under
Contingency Fund of India to deal with any emergency situation. The Parliament enacted the contingency
fund of India Act in 1950. Any expenditure incurred from this fund requires a subsequent authorization
from the Parliament. And the corpus has to be replenished with the same amount later.
Statement 2 is incorrect: Article 267 of the Constitution mandates formation of a corpus under
Contingency Fund of India to deal with any emergency situation. The constitution itself has not fixed the
corpus of the fund. The fund size is enhanced from time to time by the government. In 2005, the corpus of

Forum Learning Centre: Delhi - 2nd Floor, IAPL House, 19 Pusa Road, Karol Bagh, New Delhi - 110005 | Patna - 2nd floor, AG Palace, E Boring Canal Road,
Patna, Bihar 800001 | Hyderabad - 1st & 2nd Floor, SM Plaza, RTC X Rd, Indira Park Road, Jawahar Nagar, Hyderabad, Telangana 500020
9821711605 | https://academy.forumias.com | admissions@forumias.academy | helpdesk@forumias.academy
Page 37 of 38

SFG 2023 | LEVEL 1 | Test #5 – Solutions |


the fund was raised from Rs 5 crore to Rs 500 crore. The government enhanced the Contingency Fund of
India from Rs 500 crore to Rs 30,000 crore through the Finance Bill 2021. The fund can be increased
through a Finance Bill when the Parliament is in the session. Or through Ordnance if the House is not in
session.
Statement 3 is incorrect: The Union finance secretary (not the cabinet secretary) holds the fund on
behalf of the President of India. The fund size is enhanced from time to time by the government. President
can make advances out of it to meet unforeseen expenditure pending its authorization by the Parliament.
Source: https://www.business-standard.com/podcast/finance/what-is-the-contingency-fund-of-
india-122011000060_1.html
Indian polity by M Laxmikanth pdf 6th edition. Chapter 22- parliament. Page no-546.

Q.49) Consider the following statements with respect to Parliamentary Forums in India:
1. They are constituted at the start of Budget Session each year.
2. They seek to provide a platform to the Members of Parliament to have interactions with subject experts
on a matter.
3. The Speaker of Lok Sabha is the ex-officio Vice-President of all the parliamentary forums.
Which of the statements given above is/are correct?
a) 1 only
b) 2 and 3 only
c) 2 only
d) 1, 2 and 3

Ans) c
Exp) Option c is the correct answer.
The first Parliamentary Forum on Water Conservation and Management was constituted in the year 2005.
At present, there are eight Parliamentary forums. Each Forum consists of not more than 31 members
(excluding the President, Co-President and Vice-Presidents) out of whom not more than 21 are from the
Lok Sabha and not more than 10 are from the Rajya Sabha.
Statement 1 is incorrect: They are permanent forums constituted by the Parliament to discuss specific
issues of public importance. They are not constituted every year at the start of Budget Session. Also, the
duration of the office of members of the forum is co-terminus with their membership in the respective
Houses. A member may also resign from the forum by writing to the Speaker/Chairman
Statement 2 is correct: It seeks to provide a platform to the members to have interactions with the
ministers concerned, experts and key officials from the nodal ministries with a view to have a focused
and meaningful discussion on critical issues with a result-oriented approach for speeding up the
implementation process.
Statement 3 is incorrect: The Speaker of Lok Sabha is the ex-officio President of all the Forums except
the Parliamentary Forum on Population and Public Health wherein the Chairman of Rajya Sabha is the ex-
officio President and the Speaker is the ex-officio, Co-President. The Deputy Chairman of Rajya Sabha, the
Deputy Speaker of Lok Sabha, the concerned Ministers and the Chairman of Departmentally-Related
Standing Committees are the ex-officio Vice Presidents of the respective Forums.
Knowledge Base:
The first Parliamentary Forum on Water Conservation and Management was constituted in the year 2005.
Subsequently, seven more Parliamentary forums were constituted. At present, there are eight
Parliamentary forums namely:
1) Parliamentary Forum on Water Conservation and Management (2005)
2) Parliamentary Forum on Youth (2006)
3) Parliamentary Forum on Children (2006)

Forum Learning Centre: Delhi - 2nd Floor, IAPL House, 19 Pusa Road, Karol Bagh, New Delhi - 110005 | Patna - 2nd floor, AG Palace, E Boring Canal Road,
Patna, Bihar 800001 | Hyderabad - 1st & 2nd Floor, SM Plaza, RTC X Rd, Indira Park Road, Jawahar Nagar, Hyderabad, Telangana 500020
9821711605 | https://academy.forumias.com | admissions@forumias.academy | helpdesk@forumias.academy
Page 38 of 38

SFG 2023 | LEVEL 1 | Test #5 – Solutions |


4) Parliamentary Forum on Population and Public Health (2006)
5) Parliamentary Forum on Global Warming and Climate Change (2008)
6) Parliamentary Forum on Disaster Management (2011)
7) Parliamentary Forum on Artisans and Crafts-people (2013) 8. Parliamentary Forum on Millennium
Development Goals (2013)
Source: M. Laxmikant 6th Edition.pdf

Q.50) Which one of the following agencies/organizations has released the National Multidimensional
Poverty Index (MPI)?
a) Department of Health and Research
b) Oxfam India
c) United Nations Development Programme (UNDP)
d) NITI Aayog

Ans) d
Exp) Option d is the correct answer.
This question is based on the following articles “One out of every two Bihar households is
multidimensionally poor: NITI Aayog” published in Down To Earth on 27th November 2021.
Option d is correct. India’s first National MPI (multidimensional poverty index) has been released by Niti
Aayog. National MPI seeks to measure poverty across its multiple dimensions and in effect complements
existing poverty statistics based on per capita consumption expenditure.
Methodology: The index has used the globally accepted and robust methodology developed by the Oxford
Poverty and Human Development Initiative (OPHI) and the United Nations Development Programme
(UNDP) for Global MPI.
Indicators: The index is based on three equally weighted dimensions – health, education and standard of
living – which in turn are represented by 12 indicators.
Key findings of the National MPI:
1) Multidimensionally Poor: Around 25.01% of the Indian population is multidimensionally poor. Bihar has
the maximum percentage of the population living in poverty among all the States and the Union
Territories, with over 50% of the population in the State identified as “multidimensionally poor”.
2) Poverty Criteria: The report took a person spending less than Rs 47 a day in cities and one spending
less than Rs 32 a day in villages as poor.
3) Healthy Nutrition: Some 37.6% of Indian households are deprived of healthy nutrition levels.
4) Child and Adolescent Mortality: Some 2.7% households have reported child and adolescent mortality.
A household is deprived if any child or adolescent under 18 years of age has died in the household in the
five-year period preceding the survey.
5) Education: At least 13.9% households have a member aged 10 years or older who has not completed six
years of schooling. At least 6.4% of households have a school-aged child not attending school up to the
age at which he/she would complete class 8.
Source: One out of every two Bihar households is multidimensionally poor: NITI Aayog-ForumIAS Blog

Forum Learning Centre: Delhi - 2nd Floor, IAPL House, 19 Pusa Road, Karol Bagh, New Delhi - 110005 | Patna - 2nd floor, AG Palace, E Boring Canal Road,
Patna, Bihar 800001 | Hyderabad - 1st & 2nd Floor, SM Plaza, RTC X Rd, Indira Park Road, Jawahar Nagar, Hyderabad, Telangana 500020
9821711605 | https://academy.forumias.com | admissions@forumias.academy | helpdesk@forumias.academy

You might also like